GRAY'S ANATOMY Week 9

Lakukan tugas rumah & ujian kamu dengan baik sekarang menggunakan Quizwiz!

A(. The processus vaginalis (meaning sheathlike process) is composed of parietal peritoneum that pre- cedes the testis as it "migrates" from a position in the upper lumbar wall to a position outside the abdomen. This process usually obliterates, leaving only a distal portion that surrounds most of the testis as the tunica vaginalis. Whereas these features are typical of devel- opment in the male, females also have a processus vaginalis that extends into the labia majus, although congenital inguinal hernias are more common in males than females. The other listed structures are not involved in congenital inguinal hernias. GAS 293; N 257; McM 272, 275)

1 A 1-year-old girl is admitted to the hospital with a palpable mass in her labia majora. Radiographic examination reveals that a loop of intestine has herniated into the visibly enlarged labium majus. This condi- tion is due to failure of the processus vaginalis to close off. From which of the following tissue layers is the processus derived? A. Parietal peritoneum B. Extraperitoneal tissue C. Transversalis fascia D. Dartos fascia E. Internal abdominal oblique aponeurosis

A(. The medial meniscus is firmly attached to the medial (tibial) collateral ligament. Damage to the medial collateral ligament often causes concomitant damage to the medial meniscus because of this rela- tionship. The anterior cruciate ligament lies inside the knee joint capsule but outside the synovial cavity. It is taut during extension of the knee and may be torn when the knee is hyperextended. If this were damaged along with the medial meniscus and medial cruciate ligament, an "unhappy triad" (of O'Donoghue, or Donahue, both spellings are correct; also called a "blow knee") injury would result. The lateral menis- cus is not attached to the medial collateral ligament but receives muscular attachment to the popliteus muscle. The posterior cruciate ligament also lies outside of the synovial cavity and limits hyperflexion of the knee. The tendon of the semitendinosus forms one third of the pes anserinus, with the tendons of the sartorius and gracilis making up the other two thirds. The pes anserinus (goose foot) is located at the medial border of the tibial tuberosity, and a portion can be used for surgical repair of the anterior cruciate ligament. GAS 607-609; N 495; McM 332)

10 A 19-year-old football player was hit on the lateral side of his knee just as he put that foot on the ground. Unable to walk without assistance, he is taken to the hospital. An MRI examination reveals a torn medial collateral ligament. Which structure would most likely also be injured due to its attachment to this ligament? A. Medial meniscus B. Anterior cruciate ligament C. Lateral meniscus D. Posterior cruciate ligament E. Tendon of the semitendinosus

A(. When the anterior cruciate ligament is torn, the tibia can be slightly displaced anteriorly from the area of the knee joint by pulling firmly with both hands upon the leg, with the patient in a seated posi- tion. This is a positive anterior drawer sign. GAS 612-615; N 496; McM 322)

100 A 50-year-old man is admitted to the emergency department after a car crash. An MRI examination reveals an injured anterior cruciate ligament. Physical examination reveals a positive drawer sign. Which of the following signs is expected to be present during physical examination? A. The tibia can be slightly displaced anteriorly B. The tibia can be slightly displaced posteriorly C. The fibula can be slightly displaced posteriorly D. The fibula can be slightly displaced anteriorly E. The tibia and fibula can be slightly displaced anteriorly

B(. The superficial inguinal nodes are located near the saphenofemoral junction and drain the supe- rior thigh region. The vertical group receives lymph from the superficial thigh, and the horizontal group receives lymph from the gluteal regions and the anterolateral abdominal wall. The deep inguinal lie deep to the fascia lata and receive lymph from deep lymph vessels (popliteal nodes). The external and internal iliac nodes first receive lymph from pelvic and perineal structures (GAS Fig. 6-38). GAS 570; N 472; McM 368)

109 A 42-year-old man is bitten on his posterior thigh by a dog. The superficial wound is sutured in the emer- gency department. Four days later the patient returns to the hospital with high fever and swollen lymph nodes. Which group of nodes first receives lymph from the infected wound? A. External iliac B. Vertical group of superficial inguinal C. Deep inguinal D. Horizontal group of superficial inguinal E. Internal iliac

B(. The right primary bronchus is shorter, wider, and more vertical than the left main bronchus. When a foreign body is aspirated, it is more likely to enter the right main bronchus (although in some cases the foreign body enters the left bronchus). Pulmonary vascular resistance is not related to the question. The right lower lung lobe does not have poorer venous drainage than the other lobes. GAS 172-174; N 199; McM 198)

100 A 51-year-old woman with a history of brain tumor and associated severe oropharyngeal dysphagia develops right lower lobe pneumonia after an episode of vomiting. Which of the following is the best reason that this type of aspiration pneumonia most commonly affects the right lower lung lobe? A. Pulmonary vascular resistance is higher in the right lung than the left lung B. The right main bronchus is straighter than the left main bronchus C. The right main bronchus is narrower than the main bronchus D. The right main bronchus is longer than the left main bronchus E. The right lower lung lobe has poorer venous drainage than the other lobes

E(. One important function of the deltoid liga- ment is the prevention of excessive extension of the ankle. The ligament is so strong that excessive ever- sion can cause the medial malleolus to be pulled off (an avulsion fracture) rather than tearing the deltoid ligament. GAS 638; N 514; McM 349 Deltoid ligament prevents eversion not extension of the ankle.)

101 A 23 year old male basketball player injured his foot during training and is admitted to the emergency department. An MRI examination reveals a hematoma around the medial malleolus. Upon physical examina- tion the patient shows excessive eversion of his foot. Which of the following ligaments most likely has a tear? A. Plantar calcaneonavicular (spring) B. Calcaneofibular C. Long plantar D. Short plantar E. Deltoid

E(. The ankle jerk reflex involves S1 and S2 levels. L2 to L4 are involved in the patellar reflex. L5 is not a component of a deep tendon reflex. GAS 621; N 527; McM 345)

103 A 72-year-old woman is admitted to the emer- gency department after an episode of stroke. During neurologic examination the patient shows no response to the ankle reflex test. Which of the following nerve roots is responsible for this reflex? A. L2 B. L3 C. L4 D. L5 E. S1

B(. The femoral triangle is the best place to palpate the femoral pulse. It is bounded by the sarto- rius muscle laterally, adductor longus medially, and the inguinal ligament superiorly. It contains the femoral vein, artery, and nerve (from medial to lateral, respectively). The adductor canal lies deep between the anterior and medial compartments of the thigh and therefore cannot be palpated. The popliteal fossa is the fossa at the back of the knee and contains the popliteal artery and vein, tibial nerve, and common fibular (peroneal) nerve. The femoral pulse cannot be palpated here. The inguinal canal is in the pelvis and is in communication with the anterior abdominal wall. It contains the spermatic cord in males and round ligament of the uterus in females. GAS 572, 671; N 488; McM 320)

105 A 49-year-old man is admitted to the emergency department with a cold and pale foot. Physical exami- nation reveals that the patient suffers from peripheral vascular disease; his popliteal artery is occluded and no pulse is felt upon palpation. What is the landmark to feel the pulse of the femoral artery? A. Adductor canal B. Femoral triangle C. Popliteal fossa D. Inguinal canal E. Pubic symphysis

A(. If the femoral artery is occluded, the descending branch of the lateral circumflex femoral will provide collateral circulation to the thigh. The descending genicular artery is a branch of the femoral and therefore would also be occluded. The medial circumflex femoral artery is a proximal branch of the deep femoral artery and supplies part of the head of the femur. The first perforating branch of the deep femoral artery supplies a small portion of the muscles of the posterior thigh. Finally, the obturator artery supplies a very small artery and vascularizes only the most proximal part of the head of the femur and usually only during the early years of life. GAS 561, 566, 582; N 499; McM 322)

106 A 49-year-old man is admitted to the emergency department complaining that he has difficulties walking. Physical examination reveals that the patient suffers from peripheral vascular disease. An ultrasound exami- nation reveals an occlusion of his femoral artery at the proximal portion of the adductor canal. Which of the following arteries will most likely provide collateral circulation to the thigh? A. Descending branch of the lateral circumflex femoral B. Descending genicular C. Medial circumflex femoral D. First perforating branch of deep femoral E. Obturator artery

A(. In a femoral hernia, abdominal contents are forced through the femoral ring, which is just lateral to the lacunar ligament (of Gimbernat) and just medial to the femoral vein. The femoral vein would be found immediately lateral to the femoral hernia. This is correct in most cases because in the majority of people, the femoral vein is found more medial to both the femoral artery and nerve in the femoral triangle. The adductor longus muscle as well as the pectineus muscle would be found deep and medial to the hernia. GAS 302, 572; N 488; McM 225)

107 A 34-year-old man is lifting heavy weights while doing squats. Unfortunately, while making a maximal effort, he drops the weight and immediately grabs at his upper thigh, writhing in pain. The man is admitted to the emergency department and during physical examination is diagnosed with a femoral hernia. What reference structure would be found immediately lateral to the herniated structures? A. Femoral vein B. Femoral artery C. Pectineus muscle D. Femoral nerve E. Adductor longus muscle

A(. The femoral vein lies medial to the femoral artery in the femoral sheath. The femoral sheath is broken into three compartments: lateral, intermediate, and medial. The lateral compartment contains the femoral nerve. The medial compartment encloses the femoral canal and consists of lymphatic tissue and a lymph node, plus areolar tissue. The intermediate contains the femoral vein. GAS 568, 583; N 488; McM 320)

108 A 25-year-old man, an intravenous drug abuser, had been injecting himself with temazepam (a power- ful intermediate acting drug in the same group as diaz- epam (Valium) and heroin for 5 years, leaving much residual scar tissue over points of vascular access. The patient is admitted to the emergency department for a detoxi cation program requiring an intravenous infu- sion. The femoral veins in his groin are the only acces- sible and patent veins for intravenous use. Which of the following landmarks is the most reliable to identify the femoral veins? A. The femoral vein lies medial to the femoral artery. B. The femoral vein lies within the femoral canal. C. The femoral vein lies lateral to the femoral artery. D. The femoral vein lies directly medial to the femoral nerve. E. The femoral vein lies lateral to the femoral nerve.

D(. The great saphenous vein is commonly used in coronary artery bypass grafts. Because branches of the saphenous nerve cross the vein in the distal part of the leg, the nerve can be damaged if the vein is stripped from the ankle to the knee. Stripping the vein in the opposite direction can protect the nerve and lessen the postoperative discomfort of patients. The saphenous nerve is responsible for cutaneous inner- vations on the medial surface of the leg and the medial side of the foot. Injury to this nerve will result in a loss of sensation and also can create chronic dysesthesias in the area. The common fibular (pero- neal) nerve bifurcates at the neck of the fibula into the superficial and deep fibular (peroneal) nerves, which continue on to innervate the lateral and ante- rior compartments of the leg, respectively. These nerves are lateral and therefore not associated with the great saphenous vein. The lateral sural nerve is a cutaneous nerve that arises from the junction of branches from the common fibular (peroneal) nerve and tibial nerve and innervates the skin on the pos- terior aspect of the leg and lateral side of the foot. This nerve is often harvested for nerve grafts else- where in the body. The tibial nerve is a terminal branch of the sciatic nerve that continues deep in the posterior compartment of the leg. GAS 550, 568, 603, 659; N 525; McM 338)

11 A 49-year-old man underwent a coronary bypass graft procedure using the great saphenous vein. Post- operatively, the patient complains of pain and general lack of normal sensation on the medial surface of the leg and foot on the limb from which the graft was harvested. Which nerve was most likely injured during surgery? A. Common fibular (peroneal) B. Superficial fibular (peroneal) C. Lateral sural D. Saphenous E. Tibial

D(. The fibularis (peroneal) brevis muscle origi- nates from the lateral lower two thirds of the shaft of the fibula and inserts on the tubercle at the base of the fifth metatarsal. Any injury to this area will affect this muscle. Patients will present with a weakness in the eversion of the foot. Fibularis (peroneus) longus, extensor hallucis longus and tibialis anterior all insert on the medial side of the foot and will not be affected in this patient. The gastrocnemius inserts via the Achilles tendon to the posterior surface of the calcaneus. GAS 628-630; N 506; McM 341)

110 During a battle, a 19-year-old soldier is shot in the lateral aspect of the right foot by a bullet that ricocheted off a building. The soldier is taken to a field hospital. A radiograph of the foot reveals that the base of the fifth metatarsal was completely obliterated. Which of the following muscles is most likely affected by this injury? A. Tibialis anterior B. Fibularis (peroneus) longus C. Gastrocnemius D. Fibularis (peroneus) brevis E. Extensor hallucis longus

D(. Gynecomastia is the abnormal growth of mammary glands in males. Polythelia refers to super- numerary, or extra, nipples. Polymastia refers to supernumerary breasts. Amastia refers to the absence of breasts. GAS 130-133, 139-141; N 181; McM 179)

110 A 42-year-old man is diagnosed with liver and pancreatic disease as a result of alcoholism. During physical examination it is noted that he has abnormal enlargement of his mammary glands, as a secondary result of his disease process. Which of the following clinical conditions will most likely describe this case? A. Polythelia B. Supernumerary breast C. Polymastia D. Gynecomastia E. Amastia

A(. The obturator nerve is a branch of the lumbar plexus that originates from L2 to L4. It descends medial to the psoas on the posterior abdomi-nal wall into the pelvis where it runs along the lateral wall of the lesser pelvis, above and anterior to the obturator vessels. It enters into the medial thigh via the obturator canal (an opening above the obturator membrane) to supply the obturator externus muscle and the adductors of the thigh. The femoral nerve innervates the anterior compartment of the thigh. The inferior gluteal innervates the gluteus maximus muscle, while the superior gluteal innervates the gluteus minimus and medius. The tibial nerve inner- vates the posterior compartment of the lower limb. GAS 486, 500, 563-565; N 484; McM 261)

111 A 29-year-old woman is involved in a car crash and is taken to the emergency department. Radiographs reveal a fracture of her pelvis. During healing of the pelvic fracture, a nerve becomes entrapped in the bone callus. Musculoskeletal examination reveals an inability to adduct the thigh. Which of the following nerves is most likely affected? A. Obturator B. Femoral C. Inferior gluteal D. Superior gluteal E. Tibial

A(. The common fibular (peroneal) nerve is a branch of the sciatic nerve. It descends on the lateral side of the popliteal fossa before winding around the head of the fibula. It then divides into superficial and deep nerves that supply the lateral and anterior com- partments of the leg respectively. Due to its superficial course, it is easily injured in patients with long leg casts (which run from just below the knee). The nerve supplies the dorsiflexors of the leg, the skin of the first web space (via the deep fibular), the evertors of the foot, and the skin of the lateral side of the leg and dorsum of the foot (via the superficial fibular). GAS 617, 630, 632; N 529; McM 320)

112 A 29-year-old man is brought to the physician for removal of a cast from his left leg. He had sustained a fracture of the left lower extremity 6 weeks prior which was immobilized in a cast that extended from just below the knee to the foot. At the time of injury, there was severe pain but normal strength in the extremity. When the cast was removed, physical examination showed a pronounced left foot drop with paresthesia and sensory loss over the dorsum of the left foot and lateral leg. Injury to which nerve is the most likely cause? A. Common fibular (peroneal) B. Superficial fibular (peroneal) C. Deep fibular (peroneal) D. Sciatic E. Tibial

C(. The lymphatic drainage of the foot follows its venous drainage. The small saphenous vein drains the lateral side of the foot and the posterolateral leg. It drains into the popliteal vein in the popliteal fossa. Therefore a lesion on the lateral side of the foot will drain to the popliteal nodes in the popliteal fossa. GAS 570-571; N 472; McM 359)

113 A 12-year-old boy is brought to the physician by his father because of redness and swelling of his left foot for 24 hours. Three days earlier he had scraped his foot while wading in a drainage ditch. Examination of the foot shows a purulent abrasion with edema, ery- thema, and tenderness on the lateral side. Infection will most likely spread from the lateral side of the foot to the regional lymph nodes in which area? A. Lateral surface of the thigh B. Medial malleolus, posteriorly C. Popliteal fossa D. Sole of the foot E. Superficial inguinal area

E(. The posterior cruciate ligament runs from the posterior aspect of the intercondylar area of the tibia to the medial wall of the intercondylar fossa. It prevents posterior displacement of the tibia relative to the femur. This is usually tested with the posterior drawer test, in which the physician pushes the tibia backward while the knee is flexed in an attempt to displace it posteriorly. This is called the positive pos- terior drawer sign. The anterior cruciate ligament pre- vents anterior displacement of the tibia on the femur. The medial and lateral collateral ligaments stabilize the medial and lateral sides of the knee joint, respec- tively. The medial meniscus is an intracapsular fibro- cartilage that improves the articulation of the femur and tibia. GAS 612-615; N 496; McM 332)

114 A 22-year-old soccer player collides with one of her teammates. During examination on the field, the posterior drawer test was performed and the tibia moved backward in relation to her femur. Injury to which structure is confirmed by performing this test? A. Anterior cruciate ligament B. Lateral collateral ligament C. Medial collateral ligament D. Medial meniscus E. Posterior cruciate ligament

B(. The deep fibular (peroneal) nerve is a branch of the common fibular (peroneal) nerve. It is mainly a motor nerve that innervates the anterior compart- ment of the leg. Its only cutaneous innervation is to the skin of the first web space. The saphenous nerve innervates the medial side of the leg and foot. The cutaneous branch of superficial fibular (peroneal) nerve innervates the anterior part of the lower leg and the dorsum of the foot. The sural nerve innervates the lateral side of the leg and foot. GAS 632; N 529; McM 347)

115 A 16-year-old boy presents to the emergency department with a fracture of the first and second toes of his right foot. He received an anesthetic injection in the first web space of his foot, to permit easy manipulation and correction. Which nerve was blocked by the anesthesia? A. Saphenous B. Cutaneous branch of deep fibular (peroneal) C. Cutaneous branch of superficial fibular (peroneal) D. Sural E. Common fibular (peroneal)

B(. The gluteal region can be divided into quad- rants by two lines positioned using palpable bony landmarks. One line runs inferiorly from the highest point of the iliac crest. The second line runs horizon- tally midway between the iliac crests and the ischial tuberosity. This divides the gluteal region into four quadrants. The sciatic nerve runs through the lower medial quadrant and must be avoided during intra- gluteal injections. GAS 581, 663; N 490; McM 316)

116 During an interview, a 30-year-old man who is a psychiatric patient suddenly becomes aggressive. In order to calm him down, the patient is given an intramuscular injection in the upper lateral quadrant of the buttock. The injection is given at this specific location to prevent damage to which of the following nerves? A. Lateral femoral cutaneous B. Sciatic C. Superior gluteal D. Obturator E. Inferior gluteal

B(. Ligaments act to prevent excessive move- ment of joints. When a joint is forced into a position, that ligament is stretched and will be tender or rupture if the force is severe enough. Inversion is when the sole of the foot is turned medially and therefore will stretch ligaments that oppose this action. The calca- neofibular ligament is on the lateral side and stretches between the fibula and the calcaneous. It is the only ligament that would be damaged during such an action. The calcaneonavicular and long and short plantar ligaments are located on the plantar surface of the foot and will not be damaged during inversion injuries. The deltoid ligament is located medially and will not be affected. GAS 638-647; N 514; McM 349)

119 A 16-year-old teenage girl suffered an inversion sprain of her ankle during dance class. Physical examination in the clinic most likely reveals severe tender- ness over which ligament? A. Calcaneonavicular (spring) B. Calcaneofibular C. Long plantar D. Short plantar E. Deltoid

B(. The calcaneofibular ligament is a round cord that passes posteroinferiorly from the tip of the lateral malleolus to the lateral surface of the calcaneus. A forced inversion of the foot can result in tearing of the calcaneofibular ligament and sometimes the anterior talofibular ligament as well. Both of these ligaments act to stabilize the foot and prevent an inversion injury. The plantar calcaneonavicular ligament sup- ports the head of the talus. The long plantar ligament passes from the planter surface of the calcaneus to the groove on the cuboid and is important in maintain- ing the longitudinal arch of the foot. The short plantar ligament is located deep (superior) to the long plantar ligament and extends from the calcaneus to the cuboid and is also involved in maintaining the longitudinal arch of the foot. The deltoid (medial ligament of the ankle) attaches proximally to the medial malleolus and fans out to reinforce the joint capsule of the ankle. GAS 638; N 514; McM 349)

12 A 22-year-old football player is admitted to the hospital with pain and swelling over the lateral aspect of the ankle. The emergency department doctor diag- noses an inversion sprain. Which ligament was most likely injured? A. Plantar calcaneonavicular (spring) B. Calcaneofibular C. Long plantar D. Short plantar E. Deltoid

A(. The patellar reflex causes extension of the knee and is produced by the quadriceps muscle group which consist of; biceps femoris and vastus lateralis, medialis and intermedius. Quadratus femoris is a lateral rotator of the thigh. The sartorius is a flexor of the hip and knee, and the pectineus is an adductor and flexor of the hip. GAS 586, 606; N 493; McM 323)

120 A 58-year-old man visited his physician for his annual check-up. Physical examination reveals a hyper reflexive patellar reflex. Which muscle(s) contribute(s) to the tendon that is struck when testing this reflex? A. Quadriceps femoris B. Quadratus femoris C. Sartorius D. Pectineus E. Biceps femoris

B(. The lateral femoral circumflex artery is a branch of the femoral artery close to the hip joint. It gives a branch that runs down the lateral aspect of the thigh and joins the genicular anastomosis via the superior lateral genicular artery. The medial circum- flex femoral artery does not provide any branches that descend toward the knee. The anterior and posterior tibial arteries are the terminal branches of the popli- teal artery and would not receive any blood if the popliteal is damaged. The fibular (peroneal) artery is a branch of the posterior tibial artery (GAS Fig. 6-50). GAS 566, 582; N 499; McM 220)

121 A 37-year-old unconscious man is rushed to the emergency department after being retrieved from a motor vehicle crash. On physical examination bruising and obvious deformity is seen over his left knee joint. Radiological studies showed a posteriorly dislocated supracondylar fracture with severe compression of the popliteal artery. Which of the following arteries would ensure adequate blood supply to the leg and foot in this patient? A. Medial femoral circumflex B. Lateral femoral circumflex C. Anterior tibial artery D. Posterior tibial artery E. Fibular (peroneal) artery

B(. The obturator nerve is responsible for inner- vation of the thigh adductors which form the medial compartment of the thigh. The femoral nerve inner- vates the anterior compartment and is responsible for extension of the knee. Common fibular (peroneal) nerve supplies the anterior and lateral compartments of the leg while the tibial nerve supplies the posterior compartments of the leg and thigh. The common fibular (peroneal) and tibial nerves are branches of the sciatic nerve. GAS 486, 500, 563-565; N 488; McM 261)

122 After a revascularization procedure involving the common iliac artery, a 68-year-old man has difficulty walking. Nerve conduction studies reveal decreased activity in the nerve that innervates the adductors of the thigh. Which nerve is this? A. Femoral B. Obturator C. Common fibular (peroneal) D. Tibial E. Sciatic

A(. The skin of the anterior medial thigh and medial leg and foot is supplied by the femoral nerve. The saphenous nerve is a branch of the femoral and only supplies the medial leg and foot. The obturator supplies the medial and medial posterior aspect of the thigh. The tibial nerve supplies the skin of the pos- terolateral leg, lateral ankle and foot and sole of the foot. The fibular (peroneal) nerve supplies the skin over the lateral aspect of the leg and dorsal aspect of the foot. GAS 545-549, 604; N 525; McM 320)

123 A 23-year-old woman was taken to the emergency department after being involved in a head-on collision with a truck. On physical examination a hematoma was seen in the medial thigh. A CT scan revealed a fracture of the femur with a ruptured femoral artery. She was taken to the operating room for repair of the damaged structures. Two days postoperatively during physical examination the patient has loss of sensation to the anterior medial thigh and medial side of her leg and foot. Branches of which of the following nerves were most likely injured in the repair of the fracture? A. Femoral B. Saphenous C. Obturator D. Tibial E. Fibular (peroneal)

A(. Dorsalis pedis pulse is palpated at the prom- inent arch of the top of the foot between the first and second metatarsal bones between the tendon of the extensor hallucis longus and extensor digitorum longus for the second toe. GAS 658; N 517; McM 348)

126 A 50-year-old diabetic man presents for a routine wellness checkup. During physical examination it is noted that he has paraesthesia in a classic glove and stocking distribution. The physician decides on a com- plete peripheral vascular system examination, which includes palpating the pulse of the dorsalis pedis. Where can the dorsalis pedis pulse be palpated? A. Between the tendons of extensor hallucis and extensor digitorum longus on the dorsum of the foot B. Superior to flexor hallucis longus just distal to the tarsal tunnel C. Inferolateral to the pubic symphysis and medial to the deep dorsal vein of the penis D. 2 cm anterior to the medial malleolus E. 2 cm posterior to the medial malleolus

A(. Gracilis due to its shape, size, and more importantly the nature of neurovascular supply is used very commonly in reconstructive surgery as a free functioning autograft. Also, the other adductors of the thigh compensate for the absence of the graci- lis. For similar reasons the remaining muscles are not good candidates during reconstructive surgery of the upper limb. GAS 594; N 488; McM 323)

127 A 43-year-old victim of a drunk driving car crash is undergoing reconstructive arm surgery. The surgeon performs an autograft using a weak adductor of the leg located super cially on the medial side of the thigh. Which muscle is most likely being harvested to perform this reconstruction? A. Gracilis B. Sartorius C. Rectus femoris D. Vastus lateralis E. Vastus medialis

A(. Any super cial in ammation in the gluteal region drains into the super cial horizontal group of inguinal nodes. The vertical group drains the lower limbs, whereas deep gluteal injuries drain into the superior and inferior gluteal nodes. GAS 519; N 472; McM 368)

128 A 39-year-old woman who is a school teacher unwittingly sits on a thumbtack a student placed on her chair. Her left buttock becomes painful and inflamed. Which group of nodes will first receive lymph from the infected wound? A. Superficial horizontal group B. Superficial vertical group C. Superior and inferior gluteal nodes D. External iliac E. Deep inguinal

C(. The dermatome that encompasses the nipple is supplied by spinal nerve T4. In this case the herpes zoster virus is harbored in the dorsal root ganglion of T4 and can be activated to cause the characteristic rash that is distributed along the derma- tome including the nipple. GAS 139-141; N 162; McM 178)

129 A 29-year-old patient complains of severe pain radiating across her back and chest. Upon clinical examination you observe a rash characteristic of herpes zoster infection passing from her upper left back and across her left nipple. Which of the following spinal nerve roots sheds the active virus? A. Dorsal root of T3 B. Ventral root of T3 C. Dorsal root of T4 D. Ventral root of T4 E. Dorsal root of T5

E(. Because the hamstrings cross two joints and are very crucial during all phases of running, but especially during the late swing through midstance phase of running, are easily injured. Their normal action includes hip extension and knee flexion. The do not rotate the hip. GAS 598; N 482; McM 319)

129 During a 100-meter sprint a 25-year-old male Olympic athlete suddenly pulls up in discomfort and is seen to be clutching the back of his left thigh in agony. Upon further examination the athlete describes the pain as a "tearing" sensation and is unable to ex his knee. Based on these symptoms which of the following actions are affected due to this injury? A. Flexion of the hip and extension of the knee B. Extension of the hip and dorsi flexion C. Medial rotation of the hip D. Lateral rotation of the hip E. Hip extension and knee flexion

B(. The lateral circumflex femoral artery arises from the deep femoral (profunda femoris) artery of the thigh and sends a descending branch down the length of the femur to anastomose with the superior medial genicular artery and the superior lateral genic- ular artery. The medial circumflex femoral artery is responsible for supplying blood to the head and neck of the femur, and it does not anastomose with distal vessels at the knee. The first perforating artery sends an ascending branch that anastomoses with the medial circumflex femoral and the inferior gluteal artery in the buttock. The inferior gluteal artery is a branch of the internal iliac; it has important anasto- motic supply to the hip joint. The typically small descending genicular branch of the femoral artery is given off just proximal to the continuation of the femoral artery as the popliteal. GAS 589-599; N 491; McM 319)

13 A 72-year-old woman is admitted to the hospital with a painful right foot. A CT scan examination reveals a thrombotic occlusion of the femoral artery in the proximal part of the adductor canal. Which artery will most likely provide blood supply to the leg through the genicular anastomosis? A. Medial circumflex femoral B. Descending branch of the lateral circumflex femoral C. First perforating branch of the deep femoral D. Inferior gluteal E. Descending genicular branch of femoral

B(. The rough bony projection at the junction of the inferior end of the body of the ischium and its ramus is the large ischial tuberosity. Much of the body's weight rests on these tuberosities when sitting, and it provides the proximal, tendinous attachment of the posterior thigh muscles (hamstring muscles and adductor magnus). The hamstring muscles are associ- ated with hip extension and knee flexion. The adduc- tion of the hip joint will be affected slightly because the adductor magnus is affected, although the rest of the adductor muscles are intact. GAS 551-553, 575, 590, 594; N 478; McM 292)

130 During a gymnastic session, a 24-year-old woman suddenly developed pain and swelling on the right buttock. This happened following a forceful thigh movement. There is severe weakness of right hip extension and knee exion. Adduction of the thigh is also slightly weak. An avulsion fracture of the ischial tuberosity is found on a radiograph. Which of the following group of muscles has most likely involved in this process? A. Adductor brevis, adductor longus, adductor magnus, pectineus, and gracilis B. Biceps femoris, semimembranosus, semitendinosus, and adductor magnus C. Iliacus and psoas major D. Gluteus medius and gluteus minimus E. Gluteus maximus and adductor magnus F. Iliacus, psoas major, rectus femoris and sartorius

D(. The inferior gluteal nerve supplies the gluteus maximus muscle, which extends and laterally rotates the hip joint; through the iliotibial tract, it also extends the knee joint. The superior gluteal nerve supplies the gluteus medius and minimus and tensor fasciae latae muscles, which work together as medial rotators of lower limb. The nerve to the piriformis supplies the piriformis muscle, which laterally rotates the femur with hip extension and abducts the femur with hip exion. GAS 565, 575, 579; N 490; McM 318)

140 A 29-year-old construction worker falls onto some rusty wire mesh and suffers a deep laceration to his right buttock. When the ambulance arrives to transport him to the emergency department and it is noted that he has dif culty stepping up into the ambulance with his right leg. Which nerve has probably been damaged? A. Superior gluteal B. Tibial C. Common bular (peroneal) D. Inferior gluteal E. Nerve to piriformis

D(. Duchenne muscular dystrophy is a condi- tion that causes muscle weakness. It starts in child- hood and may be noticed when a child has dif culty standing up, climbing, or running, which requires extension of the hip. This patient has the classic Gower's sign. The gluteus maximus functions primar- ily between the exed and standing (straight) posi- tions of the thigh, as when rising from the sitting position, straightening from the bending position, walking uphill and up stairs, and running. GAS 575, 590, 594; N 482; McM 318)

131 A 6-year-old boy with a family history of muscu- lar disease leading to wheelchair dependency in his maternal uncles presents with difficulty in standing from the seated position. He bends forward, uses his hands to help him push up from the floor, and then straightens his knees to stand. Which of the following muscles is most likely involved by this disease process? A. Tibialis posterior and gastrocnemius B. Quadratus femoris C. Gluteus medius and gluteus minimus D. Gluteus maximus E. Hamstrings F. Iliopsoas

A(. The gluteal region (buttocks) is a common site for intramuscular injection of drugs, particularly if the volume of the injection is large. To avoid injury to the underlying sciatic nerve, the injection should be given well forward on the upper outer quadrant of the buttock (superolateral quadrant). The patient is showing the Trendelenburg gait pattern (or gluteus medius lurch), which is caused by weakness of the gluteus medius and minimus muscles. These muscles are supplied by the superior gluteal nerve (L4, L5, S1), which emerges from the greater sciatic notch above the upper border of the piriformis and imme- diately disappears beneath the posterior border of the gluteus medius and runs forward between the gluteus medius and minimus. Intramuscular injection in the upper inner quadrant (superomedially) is most likely to damage this nerve. The sciatic nerve is most likely damaged in the inferomedial quadrant of the buttock. GAS 581; N 482; McM 316)

132 A 43-year-old woman receives deep intramuscular injections for the past week for treatment of a sexually transmitted disease. She complains to her doctor that she has dif culty walking. During physical examination her right hip drops every time she raises her right foot. Which of the following injection locations will most likely correspond with the clinical presentation of this patient? A. Superomedial quadrant of the buttock B. Superolateral quadrant of the buttock C. Inferomedial quadrant of the buttock D. Inferolateral quadrant of the buttock E. Posterior thigh

D(. The anterior cruciate ligament is attached to the anterior intercondylar area and the posterior part of the medial surface of the lateral femoral condyle. Posterior displacement of the femur on the tibia is prevented by the ACL. With the knee joint flexed, the ACL prevents the tibia from being pulled anteriorly. The posterior cruciate ligament is stronger, shorter, and broader, less oblique, and prevents anterior dis- placement of the femur on the tibia. Lateral collateral ligaments are cordlike and are attached proximally to the lateral side of the head of the fibula overlapped by the tendon of biceps femoris. The medial collateral ligament is a flat band and is attached above to the medial condyle of the femur and below to the medial surface of the shaft of the tibia. It is firmly attached to the edge of the medial meniscus and consequently is more prone to be injured. The oblique popliteal ligament is a tendinous expansion derived from the semimembranosus muscle. It strengthens the poste- rior aspect of the knee joint capsule. The patellar liga- ment (tendon) connects the lower border of the patella with the smooth convexity on the tuberosity of the tibia. It represents the continuation of the quadriceps tendon. GAS 586-588, 612; N 496; McM 332)

133 A 22-year-old male professional football player is admitted to the emergency department with acute right knee pain after sustaining a kick injury to an extended leg. A radiograph and a subsequent MRI revealed that the trauma caused anterior displacement of the tibia with respect to her femur. Which of the following liga- ments was most likely injured? A. Fibular (lateral) collateral B. Tibial (medial) collateral C. Patellar D. Anterior cruciate E. Posterior cruciate F. Oblique popliteal

B(. The psoas muscle arises from the base of the transverse processes, the sides of the vertebral bodies, and the intervertebral discs, from the twelfth thoracic to the fifth lumbar vertebrae and inserted into the lesser trochanter of the femur. The sheath of the psoas retains the pus of a psoas abscess, and spinal tuber- culosis may present as a cold abscess in the groin (in the vicinity of the lesser trochanter). The psoas is enclosed in a fibrous sheath that is derived from the lumbar fascia. The sheath is not part of the lumbar fascia, but the lateral edge blends with the anterior layer of that fascia. GAS 590-591; N 485; McM 263)

134 A 51-year-old immigrant with tuberculosis is found to have large occulent masses over the lateral lumbar spine. There is a similar mass located in the ipsilateral groin. Physical examination reveals increased tenderness just medial to the ipsilateral anterior supe- rior iliac spine on palpation. This pattern of involve- ment most likely suggests an abscess tracking along which of the following muscles? A. Piriformis B. Psoas major C. Adductor longus D. Gluteus maximus E. Obturator internus

B(. The psoas muscle arises from the base of the transverse processes, the sides of the vertebral bodies, and the intervertebral discs, from the twelfth thoracic to the fifth lumbar vertebrae and inserted into the lesser trochanter of the femur. The psoas flexes the thigh at the hip joint on the trunk, or if the thigh is fixed, it flexes the trunk on the thigh, as in sitting up from a lying position. The inflamed appendix is pushed up against the peritoneum from the the con- tracted psoas. As a result it is in touch with the pari- etal peritoneum, producing acute pain. In some other cases it may retain the purulence of a psoas abscess, and spinal tuberculosis may present as a cold abscess in the groin. The psoas is enclosed in a fibrous sheath that is derived from the lumbar fascia. The sheath is not part of the lumbar fascia but the lateral edge blends with the anterior layer of that fascia. GAS 590-591; N 485; McM 264)

135 A 23-year-old man is taken to the emergency department because of anorexia, nausea, vomiting, and severe abdominal pain in the right lower quadrant. On examination, he has tenderness in the right lower quadrant with rebound tenderness. The physician suspects appendicitis. To confirm this diagnosis, the physician attempts to straighten the patient's exed thigh. This causes the patient to wince with pain. Which of the following muscles most likely caused this symptom? A. Adductor magnus B. Psoas major C. Biceps femoris D. Obturator internus E. Gluteus maximus

B(. The saphenous nerve is the longest and most widely distributed cutaneous branch of the femoral nerve; it is the only branch not from the sciatic nerve to extend beyond the knee. It gives sensory innervations to the medial aspect of the thigh, leg, and the medial planter arch. It accompanies the great saphenous vein over the medial side of the leg. The great saphenous vein is formed by the union of the dorsal vein of the great toe and the dorsal venous arch of the foot. It ascends anterior to the medial malleolus and passes posterior to the medial condyle of the femur and ends when it joins the femoral vein. GAS 604, 662; N 525; McM 329)

136 A 60-year-old man presents with pain on the medial aspect of his thigh. During physical examination he describes the pain to be constant, nonradiating and he also complains of numbness on the medial aspect of his leg and medial plantar arch. The nerve involved in this patient's numbness is closely associated with a structure with which of the following characteristics? A. Empties into the popliteal vein B. In its ascent in the medial aspect of the leg, it travels posterior to the medial condyle of the femur C. In its ascent in the medial aspect of the leg, it travels anterior to the medial condyle of the femur D. Arches posterior to the medial malleolus E. Is associated with nodes that drain to the horizontal group of inguinal nodes

C(. The deep fibular (peroneal) nerve passes deep to the extensor retinaculum and supplies the intrinsic muscles on the dorsum of the foot (extensors digitorum and hallucis longus) and the tarsal and tarsometatarsal joints. When it finally emerges as a cutaneous nerve, it is so far distal in the foot that only a small area of skin remains available for innervation: the web of skin between and contiguous sides of the first and second toes. The superficial fibular (pero- neal) nerve supplies the skin on the anterolateral aspect of the leg and divides into the medial and intermediate dorsal cutaneous nerves, which continue across the ankle to supply most of the skin on the dorsum of the foot. GAS 631-632; N 529; McM 347)

137 A 56-year-old diabetic man complains of repeated injury and ulcers to his right big toe. He also complains that he finds it difficult maintaining his shoes because the tips of the shoes around the toe area easily wear down. He also complains that for a while now, his first two toes "feel funny." He used to enjoy playing soccer on weekends but has found it difficult to be involved. Which of the following nerves is most likely affected? A. Superior gluteal nerve injury B. Inferior gluteal nerve injury C. Deep fibular (peroneal) nerve injury D. Superficial fibular (peroneal) nerve injury E. Common fibular (peroneal) nerve injury

E(. The obturator membrane is a fibrous sheet that almost completely closes the obturator foramen, leaving a small gap, the obturator canal, for the passage of the obturator nerve and vessels as they leave the pelvis to enter the medial thigh. The femoral canal is the small medial compartment for the lymph vessels. It is about 0.5 in (1.3 cm) long, and its upper opening is called the femoral ring. It has following borders: anteriorly the inguinal ligament; posteriorly the superior ramus of the pubis; medially the lacunar ligament; and laterally the femoral vein. A triangular- shaped defect in the external oblique aponeurosis lies immediately above and medial to the pubic tubercle. This is known as the superficial inguinal ring. The deep ring is an oval opening in the transversalis fascia and lies about 0.5 in (1.3 cm) above the inguinal liga- ment, midway between the anterior superior iliac spine and the pubic symphysis. Fossa ovalis, which refers to an oval opening in the superomedial part of the fascia lata of the thigh, lies 3 to 4 cm inferolateral to the pubic tubercle. GAS 441, 492, 565, 580; N 488; McM 271)

138 A 30-year-old man who is a bodybuilder presents to the physician's office complaining of pain and tingling sensation radiating down the inside of his thigh that was exacerbated upon thigh movement. A hernia through which opening would most likely cause this presentation? A. Femoral ring B. Super cial inguinal ring C. Deep inguinal ring D. Fossa ovalis E. Obturator canal

A(. Blockage of cutaneous lymphatic vessels results in edema of the skin surrounding the hair fol- licles, leading to an appearance like an orange peel (peau d'orange). Shortening of the suspensory liga- ments leads to dimpling of the overlying skin, not peau d'orange. Contraction of retinacula cutis results in retraction and inversion of the nipple and/or areola. Pectoralis major involvement has nothing to do with this condition but can result in xing the tumor rmly to the chest wall. GAS 141; N 182; McM 179)

138 A 47-year-old woman patient's right breast exhibited peau d'orange characteristics. This condition is primarily a result of which of the following occurrences? A. Blockage of cutaneous lymphatic vessels B. Shortening of the suspensory ligaments by cancer in the axillary tail of the breast C. Contraction of the retinacula cutis of the areola and nipple D. Invasion of the pectoralis major by metastatic cancer E. Ipsilateral (same side) inversion of the nipple from cancer of the duct system of the breast

C(. The deep nodes are located beneath the deep fascia (fascia cribrosa) and lie along the medial side of the femoral vein. The presence of swollen inguinal lymph nodes is an important clinical sign because swelling may indicate an infection in the lower extremities. They then drain superiorly to the external iliac lymph nodes. The superficial nodes lie in the superficial fascia below the inguinal ligament and can be divided into horizontal and vertical groups. External iliac lies along the external iliac vessels; they are arranged in groups of three (anteriorly, medially, and lateral to vessels). GAS 570-571; N 472; McM 368)

139 After suffering a deep stab wound to her posterior thigh, a 22-year-old woman presents to the emergency department. The wound is closed but the patient develops a subsequent wound infection. Which group of lymph nodes first receives drainage from this deep wound area, and would most likely be enlarged in this patient? A. External iliac B. Superficial inguinal C. Deep inguinal D. Common iliac E. Internal iliac

A(. The talocrural joint is a synovial hinge joint that connects the distal end of the tibia and fibula with the proximal end of the talus. The articulation between the tibia and the talus bears more weight than other joints. Dorsiflexion (toes pointing upward) and plantar flexion (toes pointing downward) are pos- sible. Dorsiflexion is performed by the tibialis anterior, extensor hallucis longus, extensor digitorum longus, and peroneus tertius. Plantar flexion is performed by the gastrocnemius, soleus, plantaris, peroneus longus, peroneus brevis, tibialis posterior, flexor digitorum longus, and flexor hallucis longus. The movements of inversion and eversion take place at the talocalcaneal joint. GAS 638; N 514; McM 349)

141 A 69-year-old woman, who fell down the stairs, presents to the emergency department. Radiologic imaging reveals a fracture of the talocrural (tibiotalar) joint. Which movements take place at this joint? A. Plantar flexion and dorsiflexion B. Inversion and eversion C. Plantar flexion, dorsiflexion, inversion, and eversion D. Plantar flexion and inversion E. Dorsiflexion and eversion

E(. Plantar flexion is mostly due to the gastroc- nemius and soleus muscles, which are supplied by the tibial nerve. The tibial nerve leaves the popliteal fossa by passing deep to the gastrocnemius and soleus mus- cles and lies posterior to the popliteal artery. Therefore a hematoma of the popliteal artery will also compress the nerve. Dorsiflexion of the foot is due to contraction of the muscles in the anterior compartment of the leg. GAS 621; N 504, 505; McM 330)

142 After being struck from behind by a motor vehicle, a 55-year-old man presents to the hospital with a swelling of his right knee. Imaging reveals a large hematoma of the popliteal artery compressing his tibial nerve. Upon neurologic examination which movement would likely be diminished in strength? A. Dorsiflexion of the foot B. Flexion of the thigh C. Extension of the digits D. Extension of the leg E. Plantar flexion of the foot

A(. A lateral blow to the knee could result in injury to three structures in the knee: anterior cruciate ligament, medial collateral ligament, and the medial meniscus. When all three structures are involved it is collectively called the "unhappy triad." Anterior drawer sign is due to injury to the anterior cruciate ligament and denoted by anterior displacement of the tibia in relation to femur, similar to pulling out a drawer from a desk. GAS 606, 673; N 496; McM 332)

143 A lateral blow to the knee during a tackle in a football game injures a 24-year-old woman. Field examination reveals an "anterior drawer sign." An MRI demonstrates injury to several structures of the knee, including her medial meniscus. Which structure might also have been injured by the tackle? A. Medial collateral ligament B. Lateral collateral ligament C. Lateral meniscus D. Posterior cruciate ligament E. Tendon of the semitendinosus

B(. The obturator membrane is a thin mem- brane that covers the obturator foramen except at its superior part. The obturator nerve exits the pelvis and enters into the medial compartment of the thigh by passing through the obturator canal alongside the obturator vessels. Traumatic injuries to the membrane will most likely lead to obturator nerve damage. The obturator nerve supplies motor innervations to the adductor muscles of the thigh (gracilis, obturator externus, adductor longus, adductor brevis and a portion of the adductor magnus). It also provides sensory innervation to the medial aspect of the thigh. Urinary and fecal incontinence is mediated by auto- nomic nerves and the pudendal nerve. Both nerves have no relationship with the obturator membrane. The gluteus medius and minimus muscles are the main hip abductors. They also stabilize the hip on the swing-side during motion. These muscles are sup- plied by the superior gluteal nerve, which leaves the pelvis through the greater sciatic foramen above the piriformis muscle. Flexors of the hip found in the anterior compartment of the thigh are innervated by the femoral nerve, which has no relationship with the obturator membrane. The sciatic nerve supplies the muscles in the posterior compartment of the thigh and also sends cutaneous innervations to the skin of the posterior thigh. It enters the posterior compartment of the thigh from the gluteal region. GAS 441, 492, 565, 580; N 488; McM 271)

145 A 30-year-old woman was admitted to the emer- gency department after being involved in a motor vehicle crash. The patient complained of pain to the right hip and knee. During physical examination, there is no deformity of the lower limb, but there is tender- ness over the right ischiopubic ramus. Pelvic radio- graphs revealed an inferiorly displaced fracture of the right superior and inferior pubic rami with dislocation of the right sacroiliac joint and pubic symphysis. The patient was referred to the orthopedics team, which had a high suspicion of rupture of the right obturator membrane. What clinical ndings are most likely to be present in this patient? A. Urinary and fecal incontinence and diminished sensation over the perineum B. Weak adduction of the hip and diminished sensation over the upper medial thigh C. Weak abduction of the hip and positive Trendelenburg sign D. Weak exion of the hip and diminished sensation over the anterior thigh and medial leg E. Weak extension of the hip and diminished sensation over the posterior thigh

A(. By placing a chest tube just superior to the upper border of the rib, the important neurovascular bundle is avoided. The bundle consists of intercostal vein, artery, and nerve running through the superior portion of the intercostal space. In each space, the vein is the most superior structure and is highest in the costal groove. The artery is inferior to the vein and the nerve is inferior to the artery and may not be protected by the costal groove on the lower border of the rib. Entrance through the middle of the space does not remove the possibility of damaging the neurovas- cular bundle. Neither passage between the internal and external intercostal muscles, nor between the intercostal muscles and the posterior intercostal mem- brane would allow entry to the pleural cavity. GAS 152-159, 160; N 186; McM 183)

146 A 34 year old man with a complaint of sharp, localized pain over the thoracic wall is diagnosed with pleural effusion. A chest tube is inserted to drain the effusion through an intercostal space. Where should the chest tube be inserted? A. Just superior to the upper border of the rib B. Just inferior to the lower border of the rib C. At the middle of the intercostal space D. Between the internal and external intercostal muscles E. Between the intercostal muscles and the posterior intercostal membrane

E(. The muscles of the limbs develop from the myotome component of the somites. The somites are condensations of paraxial mesoderm that form after the formation of the trilaminar disc. The lateral plate mesoderm gives rise to the serous membranes, cover- ings of organs and the heart. The intermediate meso- derm gives rise to the urogenital system and its accessory glands. The chondri cation centers precede the formation of the muscles, as it gives the skeletal framework. Neural crest cells give rise to dorsal root ganglia, leptomeninges, Schwann cells, sympathetic ganglia, and the chromaf n cells of the adrenal medulla. GAS 637-638; N 487)

156 A 30-year-old woman is admitted to the emer- gency department with complaints of pain to the anterior left thigh. While participating in a 100-meter race, she felt a sudden onset of pain in the anterior midthigh area and could only limp to the nish line. Physical examination revealed a swollen, tender right thigh anteriorly. Extension of the knee was limited due to pain. Ultrasonography of the area revealed a defect in the bers of the quadriceps muscle, con rmed by CT scan of the limb. Which of the following is the embryologic origin of the affected structure? A. Lateral plate mesoderm B. Dorsolateral migration of neural crest cells C. Preceded the development of chondrification centers D. Intermediate mesoderm E. Migration of cells from paraxial mesoderm

D(. A positive valgus stress test indicates injury to the medial collateral ligament. Injuries to this liga- ment usually involve the anterior cruciate ligament. The femur is usually pushed posteriorly during stair climbing, an action that is opposed by a normal ante- rior cruciate ligament. Injury to the anterior cruciate ligament results in posterior displacement of the femur in relation to the tibia with difficulty climbing stairs. Extension of the knee is done mainly by the quadriceps femoris muscle. The posterior thigh muscles provide flexion of the knee. Gravity pushes the femur forward while walking down a flight, which is stabilized by the posterior cruciate ligament, which is not damaged in this case. The patient has no dif- ficulty walking down the stairs; the posterior cruciate ligament stabilizes the knee during this action. GAS 610-615; N 496; McM 332)

147 A 40-year-old man was brought to the emergency department after being struck by a car. He complained of pain to the left knee and leg and inability to bear weight on the affected limb. On examination, there was a joint effusion of the left knee, and tenderness over the medial and lateral side of the joint. A valgus stress test was positive while the varus stress test was nega- tive. An MRI of the left knee showed complete disrup- tion of multiple ligament support structures of the knee. What other symptoms are most likely possible in this patient? A. Inability to extend the knee B. Inability to flex the knee C. Instability of the knee when walking down a flight of stairs D. Instability of the knee when walking up a flight of stairs E. Excessive extension of the knee and difficulty walking down stairs

E(. The calcaneofibular ligament is located infe- rior and just anterior to the lateral malleolus and connects the lateral malleolus to the calcaneus. The interosseous ligament between the tibia and the fibula is located medially and superior to the lateral malleo- lus. The tibionavicular ligaments are located on the medial side of the ankle joint, and the point of injury and tenderness is at the lateral side. The anterior tibiofibular ligament is located anterior to the ankle joint, away from the point of injury (GAS Fig. 6-99). GAS 638-645; N 514; McM 349)

148 A 19-year-old woman is admitted to the emer- gency department with complaints of pain and swelling to the right ankle. In a recent volleyball game she jumped to spike the ball then landed on the opponent's shoe with her right foot. She recalls hearing a loud "pop" and felt immediate pain to the ankle. She was unable to bear weight subsequently. On examination, the right ankle was swollen, with maximal tenderness inferior and anterior to the lateral malleolus. Radio- graphs of the ankle showed no fractures. Which of the following structures were most likely injured? A. Posterior talofibular ligament B. Interosseous ligament between the tibia and fibula C. Tibionavicular ligament D. Anterior tibiofibular ligament E. Calcaneofibular ligament

E(. The paraxial mesoderm develops into somites. Limb muscles develop from the ventral myotome of the somites in response to molecular signals. Embryological derivatives of the lateral plate mesoderm include the circulatory and gut wall, body wall lining, and dermis. Derivative of the neural crest cells does not include the limb muscles. Chondri ca- tion is associated with cartilage formation and not muscles. The intermediate mesoderm eventually thins out laterally and becomes the mesoderm, which gives the circulatory and gut walls, plus the lining of the body wall and dermis. GAS 592; N 479)

149 A 30-year-old woman is admitted to the emergency department with complaints of pain to the anterior left thigh. While participating in a 100-meter race, she felt a sudden onset of pain in the anterior midthigh area and could only limp to the finish line. Physical examination revealed a swollen, tender right thigh anteriorly. Extension of the knee was limited due to pain. Ultrasonography of the area revealed a defect in the fibers of the quadriceps muscle, confirmed by CT scan of the limb. Which of the following is the embryologic origin of the affected structure? A. Lateral plate mesoderm B. Dorsolateral migration of neural crest cells C. Preceded the development of chondrification centers D. Intermediate mesoderm E. Migration of cells from paraxial mesoderm

B(. The obturator nerve arises from the lumbar plexus and enters the thigh through the obturator canal. This nerve is responsible for innervation of the medial compartment of the thigh (adductor compart- ment). Injury to this nerve can result in weakened adduction and difficulty walking. The femoral nerve innervates muscles of the anterior compartment of the thigh that are responsible for hip flexion and leg extension. The sciatic nerve branches into the common fibular (peroneal) and tibial nerves. The common fibular (peroneal) nerve branches into the deep and superficial branches of the fibular (peroneal) nerve responsible for innervation of the anterior and lateral compartments of the leg, respectively. The tibial nerve innervates the muscles of the posterior compartment of the thigh and leg, which are responsible for exten- sion of the hip, flexion of the leg, and plantar flexion of the foot. GAS 486, 500, 563-565; N 485; McM 261)

15 A 56-year-old man with advanced bladder carci- noma suffers from dif culty while walking. Muscle testing reveals weakened adductors of the right thigh. Which nerve is most likely being compressed by the tumor to result in walking difculty? A. Femoral B. Obturator C. Common bular (peroneal) D. Tibial E. Sciatic

A(. Development of the upper limb begins at 26 to 27 days after fertilization with the appearance of the upper limb buds; at 33 to 36 days the hand plates are formed and the digital rays are present; at 44 to 46 days notches form between digital rays, and at 49 to 51 days webbed ngers are distinct. Separate ngers appear at 52 to 53 days. 58-60)

29 Below are embryonic changes that occur during development of the upper limbs: 1. Separate fingers 2. Limb bud development 3. Webbed fingers 4. Digital rays Which of the following most likely represents the correct order of developmental changes that occur in the upper limbs between weeks 5 and 8? A. 2-4-3-1 B. 3-4-2-1 C. 2-3-4-1 D. 4-2-3-1 E. 1-4-3-2

C(. Mesenchyme between digital rays undergoes apoptosis for the digits to form. Failure or incomplete apoptosis usually results in fused digits (syndactyly). This may involve the skin and soft tissues alone or may include the bone. Digital rays form from the hand plate. Failure of development of any digital ray results in underdevelopment of a nger or toe. Neural crest cells do not contribute to the formation of the foot. The zone of polarizing activity modulates the pattern- ing of the limb in the anteroposterior diameter. The abnormality described did not involve the phalanges, as shown by radiograph, and thus could not have been caused by faulty chondri cation. GAS 637-638; N 507; McM 335)

150 A 23-year-old woman delivered a live male infant at 37 weeks' gestation after an uneventful pregnancy. Examination of the infant revealed the right second and third toes were fused. Radiographs of the right foot indicated 14 phalanges in their correct position. After review by the pediatric orthopedic surgeon, corrective surgery for the deformity was scheduled. Which of the following embryologic conditions explains the infant's condition? A. The digital ray for the third toe did not develop B. Excessive neural crest cell migration into the foot C. Incomplete apoptosis of tissue between digital rays D. Lack of signal from the zone of polarizing activity (ZPA) E. Faulty development of chondrification centers

B(. The right superior gluteal nerve is the cor- rect choice. When a person stands on one leg or walks, the gluteus medius, gluteus minimus, and tensor muscles of the fascia latae act in synergy to stabilize the hip joint by abducting the hip (pelvic tilt). These muscles receive their innervation from the superior gluteal nerve. The abductors of the hip, as they contract to maintain the stability of the hip joint, draw the pelvis forcefully toward the weigh-bearing leg, causing the opposite side of the pelvis to tilt in that same direction. The right superior gluteal nerve innervates its ipsilateral medius, minimus, and tensor muscles of the fascia latae. Loss of these muscles results in a positive Trendelenburg sign with the pelvis dropping on the left side. GAS 565, 575; N 489; McM 318)

152 A 53-year-old woman reports difficulty in walking. Physical examination showed a positive Trendelenburg sign when she is asked to stand on her right leg. Which nerve has been compromised to produce the positive sign? A. Sciatic B. Right superior gluteal C. Left inferior gluteal D. Left superior gluteal E. Right inferior gluteal

E(. The deep fibular (peroneal) nerve along with the superficial fibular (peroneal) nerve are branches of the common fibular nerve. The deep fibular (peroneal) nerve innervates muscles of the anterior compartment of the leg dorsiflexors of the foot and the skin between the great toe and second toes, while the superficial fibular nerve innervates the lateral compartment muscles of the leg, which are evertors of the foot and the skin on most of the dorsum of the foot. If the common fibular nerve were damaged all the structures that receive innervation via this nerve will be compromising dorsiflexion and eversion. Damage to the superficial fibular nerve affects the ability to evert the foot but does not result in foot drop, making the deep fibular nerve, which innervates the dorsiflexors of the foot, the best choice as the injury describes foot drop with the ability to evert the foot conserved. The saphenous nerve is a cutaneous nerve, while the tibial nerve innervates posterior compartment muscles. GAS 617, 632; N 529; McM 331)

153 A 45-year-old man is admitted to the emergency department after a fall and subsequent leg injury. On physical examination the patient has a foot drop but eversion is unaffected. Which nerve is most likely injured? A. Tibial B. Common fibular (peroneal) C. Superficial fibular (peroneal) D. Saphenous E. Deep fibular (peroneal)

C(. The suspensory ligaments (of Cooper) are well-defined condensations of connective tissue that run from the clavicle to the dermis of the skin overlying the breast. They support and suspend the breast from the chest wall. Carcinoma of the breast produces tension on these ligaments and causes dimpling of the breast. Invasion of the axillary lymph nodes results in stagnation and fibrosis of lymph resulting in the peau d'orange appearance of the overlying skin. GAS 140-141; N 179; McM 179)

153 A 41-year-old woman is examined in the outpatient surgical clinic for a lump in her right breast. Physical examination reveals dimpling of skin of the breast over the mass. A CT scan of the breast reveals a 3-cm mass at the right upper quadrant of her right breast with multiple calcifications. The dimpling of the breast is most likely caused by invasion of the tumor into which of the following structure(s)? A. Lactiferous ducts B. Mammary and apical lymph nodes C. Suspensory ligaments D. Deltopectoral fascia E. Medial and lateral pectoral nerves

A(. All the branches of the posterior femoral cutaneous nerve are cutaneous. It arises from the dorsal divisions of the first and second and the ventral divisions of the second and third sacral nerves and travels through the greater sciatic foramen beneath the piriformis muscle to innervate the shin over the lower parts of the gluteus maximus muscle through the inferior clunial nerves and the posterior surface of the thigh and leg and perineum via its perineal branches. The lateral femoral cutaneous nerve, which innervates the lateral surface of the thigh, is an incor- rect choice; the obturator, sciatic, and femoral nerves all have both motor and sensory branches and can be eliminated, since no motor deficits were described. GAS 631-632; N 529; McM 337)

155 A 45-year-old man after being diagnosed with a posterior acetabular fracture is taken to the operating room to repair the fracture. During the neurological examination the physician notices loss of sensation to the skin of the inferior half of the buttocks, posterior, and upper medial thigh. The patient had a normal neurovascular examination preoperatively. Which of the following nerves was mostly likely damaged during the operation? A. Posterior femoral cutaneous B. Obturator nerve C. Sciatic D. Femoral E. Lateral femoral cutaneous

A(. Congenital inguinal hernias occur when a large patency of the processus vaginalis remains so that a loop of intestine herniates into the inguinal canal. A congenital hydrocele is also caused by a patent segment of a processus vaginalis lled with uid, but it does not cause an indirect hernia. Ectopic testes occur when the gubernaculum does not migrate correctly during development and the testis does not reach the scrotum, but this does not cause a hernia. Epispadias occurs when the external urethral ori ce opens onto the dorsal surface of the penis and is generally associated with exstrophy of the bladder. A rupture, or tear, of the transversalis fascia would not cause the intestines to herniate through the deep inguinal ring and therefore would not cause an indi- rect inguinal hernia. GAS 299-301; N 257; McM 223 )

157 A 3-year-old boy is admitted to the pediatric clinic with a palpable mass in the right side of his scrotum, and a preliminary diagnosis is made of a congenital, indirect inguinal hernia. Which of the following is the most likely cause of an indirect inguinal hernia in this patient? A. The deep inguinal ring opens into an intact processus vaginalis B. Congenital hydrocele C. Ectopic testis D. Epispadias E. Rupture of the transversalis fascia

A(. Knowledge of Hilton's Law would lead to this correct answer. This law in a modified form, can be remembered as "a joint is innervated by the same nerves that innervate the muscles that move that joint." A complete explanation of this law can be found in an article by Hebert-Blouin et al., Clinical Anatomy 27:548-555, 2013. The deep fibular (pero- neal) nerve is the only nerve listed that innervates muscles that move the ankle joint.)

158 A 15-year-old boy falls and injures his ankle while skateboarding. Examination in the emergency depart- ment leads to the conclusion that the ankle is mildly sprained, and it is wrapped with an elastic bandage. The boy still complains of pain in his ankle. Which of the following peripheral nerves is involved in carrying pain sensation from the ankle? A. Deep fibular (peroneal) B. Femoral C. Obturator D. Posterior femoral cutaneous E. Sural

B(. The common fibular (peroneal) nerve winds around the neck of the fibula before dividing into superficial and deep branches that go on to innervate the lateral and anterior compartments of the leg, respectively. These compartments are responsible for dorsiflexion and eversion of the foot, and injury to these nerves would result in deficits in these move- ments. The tibial nerve lies superficially in the popli- teal fossa. This nerve innervates the posterior compartment of the leg, so compression in this area would result in a loss of plantar flexion and weakness of inversion. The lateral compartment of the leg is innervated by the superficial fibular (peroneal) nerve and is mainly involved in eversion of the foot. The cutaneous branches of the superficial fibular (pero- neal) nerve emerge through the deep fascia in the anterolateral aspect of the leg and supply the dorsum of the foot. The anterior compartment of the leg is innervated by the deep fibular (peroneal) nerve and is mainly involved in dorsiflexion of the foot. The medial malleolus is an inferiorly directed projection from the medial side of the distal end of the tibia. The tibial nerve runs near the groove behind the medial malleolus, and compression at this location would result in loss of toe flexion, adduction, abduction, and abduction of the great toe. GAS 486, 550, 563-565, 607; N 529; McM 337)

16 Upon removal of a knee-high leg cast, a 15-year- old boy complains of numbness of the dorsum of his right foot and inability to dorsiflex and evert his foot. Which is the most probable site of the nerve compression that resulted in these symptoms? A. Popliteal fossa B. Neck of the fibula C. Lateral compartment of the leg D. Anterior compartment of the leg E. Medial malleolus

A(. The processus vaginalis is formed as the parietal peritoneum layer of the abdominal wall (inguinal region) evaginates through the deep ingui- nal ring and continues through the super cial ingui- nal ring. Normally, this evagination or outpouching is obliterated during development. A cyst can develop in a segment of the processus (which is also referred to as the canal of Nuck) if this processus is not oblit- erated. Congenital hydrocele would present at the base of the canal; in this case, the swelling would be in the labium majus. An ectopic uterus would present as a mass in the pelvis and not the inguinal region. A femoral hernia would be palpated below the ingui- nal ligament (usually) just medial to the femoral tri- angle. A defect of the transversalis fascia could result in in ammation in a speci c area but would not be located along the inguinal ligament because this fascial layer is located deep to the inguinal ligament. GAS 269; N 257; McM 223 )

161 A 3-year-old girl is admitted to the pediatric clinic because of a palpable right inguinal mass. An open surgical procedure is performed. Digital pressure is used to return organ contents of the hernia to the abdomen. A sac of peritoneum can be seen clearly, protruding from the internal ring. Which of the following terms is most accurate for the origin of this structure? A. A patent processus vaginalis (canal of Nuck) B. Congenital hydrocele C. Ectopic uterus D. Femoral hernia E. Rupture of the transversalis fascia

A(. The gluteus maximus is innervated by the inferior gluteal nerve, and this muscle is responsible for extension and lateral rotation of the thigh. It is the primary muscle that extends the exed hip and is used to rise from a seated position. The gluteus minimus is innervated by the superior gluteal nerve and is responsible for abduction of the thigh. Ham- string muscles are innervated by the tibial portion of the sciatic nerve, and these are responsible for exten- sion of the thigh and exion of the leg. The iliopsoas muscle is innervated by L1 and L2 and the femoral nerve, and exes the thigh. The obturator internus is innervated by the nerve to the obturator internus and is a lateral rotator of the thigh.GAS 572, 575; N 482; McM 316)

17 A 32-year-old patient received a badly placed intramuscular injection to the posterior part of his gluteal region. The needle injured a motor nerve in the area. Later, he had great dif culty rising to a standing position from a seated position. Which muscle was most likely affected by the injury? A. Gluteus maximus B. Gluteus minimus C. Hamstrings D. Iliopsoas E. Obturator internus

D(. A tension pneumothorax occurs when intrapleural air accumulates progressively, exerting positive pressure on mediastinal and intrathoracic structures. It is a life-threatening occurrence requiring rapid recognition and treatment if cardiac arrest is to be avoided. Pericardial effusion is an abnormal accu- mulation of uid in the pericardial cavity. GAS 242; N 193; McM 184 )

171 A 25-year-old woman was brought to the emer- gency department after suffering a gunshot wound to the back. During physical examination, she was found to be in cardiopulmonary distress. Vital signs: P 110 beats/min, BP 90/50, RR 32/min. The entry wound of a bullet was noted on the left thoracic area of her back. The left thorax was notably larger than the right with decreased breath sounds on auscultation and hype resonance to percussion of the left chest. A tracheal tug to the right side was noted. She was assessed as having a tension pneumothorax and the physician prepared to perform an emergency decompression of the left thorax. Between which layers will the needle have to be placed to relieve the pneumothorax? A. Between the visceral and parietal layers of the pericardium B. Between the serous and brous layers of the pericardium C. Between the mediastinal pleura and brous pericardium D. Between the parietal and visceral layers of the pleura E. Between the endothoracic fascia and parietal pleura

C(. The dermatome of spinal nerve level T10 crosses the level of the umbilicus; that of T7 is at the level of the xiphoid process. T8 and T9 dermatomes lie between the two preceding spinal nerve levels. T12 innervates the lowest portion of the rectus abdominis and overlying skin with motor and sensory supply, respectively. L1 distribution by iliohypogastric and ilioinguinal nerves supplies the suprapubic region, the pubic area, and anterior portions of the urogenital region. Pain from appendicitis is most often perceived at rst in the periumbilical region, re ecting the level of embryologic spinal nerve supply to the appendix, which is from T10. When the appendix swells and/or ruptures and contacts the body wall, somatic sensory bers of the adjacent body wall cause the apparent site of pain to shift to the lower right abdominal quadrant. GAS 289-290; N 303, 162; McM 215 )

175 A hard mass (a fecalith) in the ostium of a 27-year- old patient's appendix had led to a local infection (appendicitis) with a slightly elevated temperature and a moderate increase in WBC count. The initial pain from the infection was dull and difficult to localize, but the patient placed his hand in the periumbilical area to indicate the general area of discomfort. The region of the umbilicus receives its sensory supply, classically, from which of the following spinal nerves? A. T7 B. T8 C. T10 D. T12 E. L1

B(. The medial branch of the deep fibular (pero- neal) nerve accompanies the dorsalis pedis artery and innervates the skin between the contiguous sides of the first and second toes. The saphenous nerve is responsible for cutaneous innervation of the antero- medial aspect of the leg and foot. The superficial fibular (peroneal) nerve innervates most of the dorsum of the foot, with the exception of the area where sen- sation was lost (medial branch of deep fibular nerve). The common fibular (peroneal) nerve gives off a cuta- neous branch, the sural nerve, which innervates the lateral aspect of the leg and lateral side of the foot. GAS 630, 659-660; N 508; McM 337)

18 During the preparation of an evening meal a female medical student dropped a sharp, slender kitchen knife. The blade pierced the first web space of her foot, resulting in numbness along adjacent sides of the first and second toes. Which nerve was most likely injured? A. Saphenous B. Deep fibular (peroneal) C. Superficial fibular (peroneal) D. Sural E. Common fibular (peroneal)

C(. The unilateral area of skin innervated by the sensory bers of a single spinal nerve is called a der- matome. The 7th thoracic spinal nerves supply the area of the xiphoid process. The 10th thoracic spinal nerves supply the area around the umbilicus. The 12th thoracic spinal nerves supply the hypogastric area (GAS Fig. 4-38). GAS 289-290; N 162; McM 215)

181 A 35-year-old immune-compromised woman is suffering from herpes zoster (shingles). She presents with a severe sharp burning pain and vesicular erup- tion on the skin around the umbilicus. Which spinal nerve contributes to this dermatome? A. T8 B. T9 C. T10 D. T11 E. T12

A(. Pleuritic pain is due to in ammation of the parietal pleura which is mainly supplied by the inter- costal nerves. The phrenic nerve only supplies the central and diaphragmatic parts of the visceral pleura which are not typically affected and not amenable to nerve blocks. The vagus nerve supplies visceral effer- ents and afferents to the lungs and visceral pleura but not the parietal pleura. Likewise the cardiopulmonary nerves do not carry somatic afferent bers. The recur- rent laryngeal nerve is a branch of the vagus nerve and does not innervate the lungs. GAS 162-167; N 188; McM 182)

182 A 55-year-old man is admitted to the emergency department because of chills followed by a painful dry cough and fever for the past 3 days. The patient complains of painful breathing and upon auscultation, a pleural rub is heard when the patient breathes. A radiograph shows signs of pleurisy. The physician decides to administer lidocaine (a local anesthetic agent) to which of the following nerves? A. Intercostal nerves B. Phrenic nerve C. Vagus nerve D. Cardiopulmonary E. Recurrent laryngeal nerve

E(. The indirect inguinal hernia is the most common form of hernia and is believed to be congeni- tal in origin. The hernia sac enters the inguinal canal through the deep inguinal ring and lateral to the infe- rior epigastric vessels. The direct inguinal hernia makes up about 15% of all inguinal hernias. The sac of a direct hernia bulges directly anteriorly through the posterior wall of the inguinal canal medial to the inferior epigastric vessels. The hernia sac descends through the femoral canal within the femoral sheath, creating a femoral hernia. Acquired umbilical hernia of adults is more correctly referred to as a paraumbili- cal hernia. The hernia sac does not protrude through the umbilical scar, but through the linea alba in the region of the umbilicus. The lumbar hernia occurs through the lumbar triangle and is rare. The lumbar triangle (Petit's triangle) is a weak area in the poste- rior part of the abdominal wall. GAS 299-301; N 257; McM 225)

187 A 45-year-old man is admitted to the hospital with groin pain and a palpable mass just superior to the inguinal ligament. The patient is diagnosed with a hernia and surgical repair is performed. During the operation, the surgeon found a loop of intestine in the deep inguinal ring. Which type of hernia is this? A. Direct inguinal B. Umbilical C. Femoral D. Lumbar E. Indirect inguinal

E(. The posterior cruciate ligament is responsible for preventing the forward sliding of the femur on the tibia. The anterior cruciate ligament prevents poste- rior displacement of the femur on the tibia. The lateral collateral ligament limits extension and adduction of the leg. The medial meniscus acts as a shock absorber and cushions the articular surfaces of the knee joint. GAS 584, 606; N 496; McM 332)

19 Following an injury suffered in a soccer match, a 32-year-old woman is examined in a seated position in the orthopedic clinic. Holding the right tibia with both hands, the clinician can press the tibia backward under the distal part of her femur. The left tibia cannot be displaced in this way. Which structure was most likely damaged in the right knee? A. Anterior cruciate ligament B. Lateral collateral ligament C. Medial collateral ligament D. Medial meniscus E. Posterior cruciate ligament

C(. An indirect inguinal hernia occurs when a loop of bowel enters the spermatic cord through the deep inguinal ring (lateral to the inferior epigastric vessels). The ilioinguinal nerve runs with the sper- matic cord to innervate the anterior portion of the scrotum and proximal parts of the genitals and could be compressed during an indirect inguinal hernia. The other nerves listed are not likely to be compressed by the hernia. The iliohypogastric nerve innervates the skin of the suprapubic region. The lateral femoral cutaneous nerve innervates the skin over the lateral thigh. The subcostal nerve innervates the band of skin superior to the iliac crest and inferior to the umbilicus. The pudendal nerve innervates the musculature and skin of the perineum. GAS 301-302; N 256; McM 224)

19 A 32-year-old man is admitted to the emergency department with groin pain. Examination reveals that the patient has an indirect inguinal hernia. Which of the following nerves is compressed by the herniating structure in the inguinal canal to give the patient pain? A. Iliohypogastric B. Lateral femoral cutaneous C. Ilioinguinal D. Subcostal E. Pudendal

C(. Since the sternal angle (of Louis) is in the transverse plane the best plane to view both the ante- rior and posterior mediastinum, as well as superior mediastinal structures, would be a lateral view. Antero- posterior or posteroanterior would make it dif cult separating anterior and posterior structures. Since the sternal angle (of Louis) is in the axial plane an axial cut may miss the level completely. GAS 128-129; N 213; McM 194)

191 Chest radiographs were taken of a 70-year-old woman who complained of a cough of 3 weeks' duration but was able to ambulate. While reading the films with a medical student, the radiologist asked the student to identify the sternal angle (of Louis). Which of the following radiographic views would best demonstrate this landmark? A. Anteroposterior B. Posteroanterior C. Lateral D. Apical lordotic E. Axial

A(. The pubococcygeus muscle, especially its most medial portion, the puborectalis, is of prime importance in fecal continence. The levator ani con- sists of two major portions, the pubococcygeus and iliococcygeus, which help support pelvic viscera and resist increases in intraabdominal pressure. The puborectalis muscle is the most medial and inferior portion of the pubococcygeus. The puborectalis forms a loop around the anorectal junction, which should keep the anorectal angle to around 90 degrees; the integrity of this muscle is critical in the maintenance of fecal continence. The coccygeus and pubovesico- cervical fascia are not in direct contact with the rectum. Damage to the urogenital diaphragm can con- tribute to urinary incontinence but not fecal inconti- nence (GAS Fig. 5-7). GAS 454-456; N 336; McM 264)

21 A 55-year-old woman complains of fecal incontinence. The most likely contributing factor to such a problem is atrophy, paralysis, or dysfunction of which of the following structures? A. Pubococcygeus muscle B. Iliococcygeus muscle C. Coccygeus muscle D. Pubovesicocervical fascia E. Urogenital diaphragm

E(. The ilioinguinal and iliohypogastric nerves are located directly posterior to the kidney. Much of the sensory distribution of the iliohypogastric nerve is to the skin over the posterolateral gluteal and pubic regions. The ilioinguinal nerve supplies the upper medial thigh and, speci cally, the skin over the mons pubis and labia majora in women or skin over the root of the penis and anterior scrotum in men. The lateral cutaneous nerve of the thigh gives sensory innervation to the lateral thigh. The femoral nerve is the nerve of the anterior thigh and supplies muscles and skin of this region. The subcostal nerve runs just inferior to the 12th rib and supplies the muscles and skin of the surrounding area. GAS 398; N 253; McM 253)

210 A 68-year-old man complained of acute onset low back pain on the right with concomitant loss of sensation of the skin in the posterolateral gluteal and anterior pubic regions on the right side. During the physical examination, the patient experienced severe pain inferior to the twelfth rib at the right costovertebral angle when the physician palpated there. The physician also found that the patient had paralysis of the right internal oblique and transversus abdominis muscles. An abdom- inal CT scan revealed a tumor that originated in the right kidney, broke through the center of the posterior renal capsule and pressed on the ipsilateral quadratus lumborum muscle. Which of the following nerves was most likely compressed by the tumor? A. Subcostal B. Lateral cutaneous nerve of thigh C. Ilioinguinal D. Femoral E. Iliohypogastric

C(. The muscle that will provide the "6-pack" look is rectus abdominis, which runs between the pubic crest and the costal margin in the midline of the abdominal wall. The upper attachment is the lower border of the 7th costal cartilage and the muscle often covers the costal cartilages for a short distance anteriorly. Ribs 10-12 and the iliac crest are too far lateral for rectus abdominis to attach to it. The xiphoid process may be covered by the muscle but does not serve as an attachment. GAS 286; N 246; McM 218)

211 A 15-year-old boy who is an amateur bodybuilder has dreams of one day becoming a professional body- builder. To that end, he begins an intensive abdominal workout routine consisting of sit-ups and crunches to develop his "6-pack." The muscle that this adolescent is isolating in his workout to get the "6-pack" look attaches to which of the following skeletal areas? A. Iliac crest B. Ribs 10 to 12 C. Pubic crest D. Costal cartilages 7 to 12 E. Xiphoid process

C(. There are three major structures that pass through the diaphragm. At the level of T10 the esoph- agus will pass through while the inferior vena cava will traverse at T8 and the aorta at T12. The levels T7 and 11 are not associated with any particular struc- tures passing through the diaphragm.)

212 A 52-year-old man with a long history of alcoholic cirrhosis complained of severe dysphagia and retrosternal "burning" pain. During esophagoscopy, the endos- copist advanced the endoscope until its tip reached the esophageal hiatus of the diaphragm. At which of the following vertebral levels did the tip of the endoscope most likely end? A. T7 B. T8 C. T10 D. T11 E. T12

C(. The ascending lumbar vein connects the common iliac veins to the azygos system bilaterally and would provide an additional pathway for blood. The left common iliac will join with the right common iliac to give rise to the inferior vena cava. The azygos and hemiazygos veins drain the posterior thoracic walls of the right and left sides respectively and will receive the ascending lumbar veins in most instances. The left renal vein drains into the inferior vena cava superior to the L3 vertebra and although it receives the left gonadal vein it cannot provide an alternative pathway for blood. GAS 390-391; N 189; McM 209)

217 An abdominal computed tomography scan of a 68-year-old woman revealed complete occlusion of her inferior vena cava at the L3 vertebral level. As a result of the occlusion, which of the following veins most likely acted as collateral channels to allow blood to flow from the lower to upper parts of her body? A. Left common iliac B. Hemiazygos C. Ascending lumbar D. Azygos E. Left renal

A(. The layers of the anterior abdominal wall in the right lower quadrant from super cial to deep are; skin, Camper's fascia, Scarpa's fascia, external oblique abdominis, internal oblique abdominis, transversus abdominis, transversalis fascia, extra peritoneal fat, and peritoneum. GAS 280; N 251; McM 220)

218 A 19-year-old gang member was shot with a 9-mm bullet in his right lower quadrant. The bullet entered and traversed the anterolateral abdominal wall. The following structures form the layers of this wall. 1. Internal oblique muscle 2. Transversalis fascia 3. Transversus abdominis muscle 4. Camper fascia 5. Scarpa fascia 6. External oblique muscle Which of the following best represents the order of structures traversed by the bullet? A. 4-5-6-1-3-2 B. 4-5-2-6-1-3 C. 4-5-1-6-3-2 D. 4-2-6-1-3-5 E. 5-4-6-1-3-2

C(. The iliofemoral ligament ("inverted Y liga- ment of Bigelow") is the most important ligament reinforcing the joint anteriorly that would resist both hyperextension and lateral rotation at the hip joint. The pubofemoral ligament reinforces the joint inferi- orly and limits extension and abduction. The ischio- femoral ligament reinforces the joint posteriorly and limits extension and medial rotation. Negative pres- sure in the acetabular fossa has nothing to do with resisting hyperextension of the hip joint but does help resist dislocation of the head of the femur. The gluteus maximus muscle extends and laterally rotates the thigh and does not particularly resist hyperextension. GAS 558-561; N 474; McM 324)

22 A 72-year-old woman suffered a hip dislocation when she fell down the steps to her garage. Which of the following is most significant in resisting hyperextension of the hip joint? A. Pubofemoral ligament B. Ischiofemoral ligament C. Iliofemoral ligament D. Negative pressure in the acetabular fossa E. Gluteus maximus muscle

E(. Indirect hernias commonly result from her- niation of the intestines through the deep inguinal ring. Direct hernias penetrate the anterior abdominal wall medial to the inferior epigastric vessels through the inguinal triangle (of Hesselbach) and do not pen- etrate the deep inguinal ring. Umbilical hernias exit through the umbilicus, not the deep inguinal ring. Femoral hernias exit through the femoral ring inferior to the inguinal ligament. Lumbar hernias can pene- trate through superior (Grynfeltt) or inferior (Petit) lumbar triangles. (GAS Figs. 4-48 and 4-49) GAS 301-302; N 256; McM 225)

22 A 45-year-old man was admitted to the hospital with groin pain and a palpable mass just superior to the inguinal ligament. The patient was diagnosed with an inguinal hernia and a surgical repair was performed. During the operation the surgeon found a loop of intestine passing through the deep inguinal ring. Which of the following types of hernia was this? A. Direct inguinal B. Umbilical C. Femoral D. Lumbar E. Indirect inguinal

C(. An intracapsular femoral neck fracture causes avascular necrosis of the femoral head because the fracture damages the radicular branches of the medial and lateral circum ex arteries that pass beneath the ischiofemoral ligament and pierce the femoral neck. Until an individual reaches about 6 to 10 years of age, blood supply to the head of the femur is provided by a branch of the obturator artery that runs with the ligament of the head of the femur. Thereafter, the artery of the ligament of the head of the femur is insigni cant. Intertrochanteric fracture of the femur would not damage the blood supply to the head of the femur but would cause complications because the greater trochanter is an attachment site for several gluteal muscles. During childhood the obturator artery provides the artery of the ligament of the head of the femur. Thrombosis of the obturator artery could result in muscular symptoms, although there are several collateral sources of blood supply in the thigh. Com- minuted fracture of the extracapsular femoral neck would not ordinarily imperil the vascular supply. GAS 558-561, 676; N 491; McM 326)

23 A 75-year-old man is transported to the emer- gency department with severe pain of his right hip and thigh. A radiologic examination reveals avascular necrosis of the femoral head (Fig. 5-2). Which of the following conditions most likely occurred to produce avascular necrosis in this patient? A. Dislocation of the hip with tearing of the ligament of the head of the femur B. Intertrochanteric fracture of the femur C. Intracapsular femoral neck fracture D. Thrombosis of the obturator artery E. Comminuted fracture of the extracapsular femoral neck

D(. Gastroschisis is congenital failure of closure of the anterior abdominal wall due to incomplete closure of lateral folds. Volvulus is twisting around of the bowel and may be a result of a diverticulum. Situs inversus is when all the internal organs are situated in the opposite side of the body. GAS 265; N 251; McM 227)

239 A 25-year-old woman in the 8th month of pregnancy went to the outpatient clinic for her prenatal checkup. Ultrasound examination of the fetus showed part of the small bowel herniating into the amniotic cavity due to failure of fusion of the lateral folds in the abdominal region. Which condition is this? A. Volvulus B. Nonrotated gut C. Situs inversus D. Gastroschisis E. Ileal diverticulum

D(. The farm instrument has injured the deep fibular (peroneal) branch of the common fibular (peroneal) nerve. It is vulnerable to injury as it arises from the common fibular (peroneal) at the neck of the fibula. The muscles denervated are largely dorsi- flexors of the foot; hence, foot drop and a high step- ping gait can occur. Sensation on the dorsum of the foot is still present; therefore, the superficial branch is mostly or entirely intact, although sensation between the first and second toes would be absent. Femoral nerve injury would result in loss of knee extension. Loss of the sciatic nerve would result in loss of both the tibial and common fibular (peroneal) nerves. Because plantar flexion is still functional, the tibial nerve has not been cut. GAS 630-632, 660; N 529; McM 337)

24 A 58-year-old male farmer was accidentally struck with a scythe (a long, curved cutting blade) by another worker while they were cutting wheat. He was admit- ted to the county hospital with severe bleeding. During physical examination the doctor noted that the patient had a foot drop; sensation was present over the dorsum of the foot and the skin of the posterior calf. Which of the following nerves was injured? A. Femoral nerve B. Sciatic nerve C. Superficial fibular (peroneal) nerve D. Deep fibular (peroneal) nerve E. Common fibular (peroneal) nerve

C(. The ilioinguinal nerve supplies skin of lower inguinal region, mons pubis, anterior scrotum or labium majus, and adjacent medial thigh; inferior most internal oblique and transversus abdominis. GAS 398; N 253; McM 220)

243 A 32-year-old man visits his family physician complaining of groin pain. Examination reveals that the patient has an indirect inguinal hernia. Which nerve is most likely responsible for the pain transmission? A. Iliohypogastric B. Lateral femoral cutaneous C. Ilioinguinal D. Lumbar splanchnic E. Greater thoracic splanchnic

E(. The internal spermatic fascia is an extension of the transversalis fascia. The internal abdominal oblique muscle lies super cial to the transverse abdominal muscle and gives origin to the cremaster muscle. The cremaster muscle covers the testis and spermatic cord. It is found between the external and internal layers of the spermatic fascia. (GAS Fig. 4-47). GAS 294-300; N 257; McM 225, 265)

245 A 33-year-old man requests a vasectomy. During the procedure the urologist separates the various layers of the spermatic cord to expose the ductus deferens so that it can be ligated and cut. From which structure is the internal spermatic fascia derived? A. Internal abdominal oblique muscle B. Cremaster muscle C. External abdominal oblique muscle D. Transversus abdominis aponeurosis E. Transversalis fascia

D(. Spinal tuberculosis can spread within the sheath of the psoas major to its insertion with the iliacus upon the lesser trochanter, presenting there also with painful symptoms. The iliopsoas muscle is the principal flexor of the hip joint. Abduction of the hips is performed by the gluteus medius and minimus with assistance from short lateral rotator muscles. Extension of the hip is a function of the gluteus maximus, together with the hamstring muscles. Inter- nal rotation is performed by the adductor muscle group. GAS 368, 589; N 485; McM 326)

29 A 61-year-old female immigrant had been diagnosed with spinal tuberculosis. The woman had developed a fluctuant, red, tender bulge on one flank, with a similar bulge in the groin on the same side. This presentation is likely due to spread of disease process within the fascia of a muscle with which of the following actions at the hip? A. Abduction B. Adduction C. Extension D. Flexion E. Internal rotation

C(. The genitofemoral nerve originates from the ventral rami of L1 and L2. The "femoral" part supplies skin to the femoral triangle area, whereas the "genito" part in males travels with the spermatic cord and supplies the cremaster muscle and scrotal skin. The ilioinguinal nerve arises from L1 and supplies the skin over the root of the penis and upper part of the scrotum in the male. The iliohypogastric nerve arises from L1 (and possibly bers from T12) and supplies skin innervation over the hypogastric region and anterolateral gluteal region. The pudendal nerve pro- vides innervation to the external genitalia for both sexes but does not innervate the cremaster muscle in males. The ventral ramus of T12 is also associated with the lower portion of the anterior abdominal wall and the iliohypogastric nerve; it does not contribute to the cremasteric re ex. GAS 299-301; N 253; McM 261)

25 A 45-year-old man entered the emergency department with a complaint of severe abdominal pain. During physical examination it is observed that his cremasteric reflex is absent. Which of the following nerves is responsible for the efferent limb of the cremasteric reflex? A. Ilioinguinal B. Iliohypogastric C. Genitofemoral D. Pudendal E. Ventral ramus of T12

C(. The Achilles tendon reflex is a function of the triceps surae muscle, composed of the gastrocnemius and soleus muscles that insert on the calcaneus. The innervation is provided primarily by spinal nerve S1. The S1 root leaves the vertebral column at the S1 foramen of the sacrum, but a herniated disc at the L5-S1 intervertebral space puts the S1 root under tension, resulting in pain and possible weakness or paralysis of S1 supplied muscles, especially the plantar flexors. A disc lesion at L3-4 would affect the L4 spinal nerve (affecting foot inversion and extension); a lesion at L4-5 would cause problems with L5 (hip abduction and knee flexion). A disc lesion at S1-2 in the sacrum is improbable, unless there was lumbarization of the S1 vertebra. The gluteal crush syndrome usually occurs when a patient has been lying unconscious and unmoving on a hard surface for an extended period of time. GAS 547-548; N 484, 514; McM 345)

25 A 45-year-old man is admitted to the emergency department after experiencing a sharp pain while lifting a box of books. He told the physician that he "felt the pain in my backside, the back of my thigh, my leg, and the side of my foot." During physical examination it is observed that his Achilles tendon jerk is weakened on the affected side. Which is the most likely cause of injury? A. Disc lesion at L3-4 B. Disc lesion at L4-5 C. Disc lesion at L5-S1 D. Disc lesion at S1-2 E. Gluteal crush syndrome of the sciatic nerve or piriformis syndrome

E(. The left testicular vein joins the left renal vein, which leads to the inferior vena cava. The left testicular vein after draining the testis and epididymis joins the left renal vein, which later joins the inferior vena cava. The testicular veins drain ipsilateral struc- tures, so the right testicular vein should not be involved in the drainage of the left testicle. The right testicular vein drains directly into the inferior vena cava. The left testicular vein has no direct connection with the left common iliac vein. The median sacral vein receives blood from the sacral region and drains into the left common iliac vein. GAS 470, 516; N 310; McM 261)

250 A 38-year-old man was diagnosed with metastatic cancer of the left testes. Which of the following most likely represents the route by which the cancer spread from the left testes to the rest of the body? A. Left testicular vein; inferior vena cava; left renal vein B. Right testicular vein; left testicular vein; inferior vena cava C. Left testicular vein; left common iliac vein; inferior vena cava D. Left testicular vein; median sacral vein; inferior vena cava E. Left testicular vein; left renal vein; inferior vena cava

E(. The pubis is described as the anteroinferior part of the pelvis. The mons pubis is a fatty pad that rests above the pubis and can be palpated as a bony prominence below. The ilium articulates posteriorly with the sacrum and terminates anteriorly as the ante- rior superior iliac spine, which is located lateral to the pubic symphysis away from the midline. The coccyx is the terminal part of the vertebral column. As such, it is too posterior a structure to be palpated through the anterior abdominal wall. The sacrum is formed by the fusion of 5 sacral vertebrae and similar to the coccyx, it cannot be palpated through the anterior abdominal wall. The ischium makes up the posterior inferior part of the pelvic bone. Its most prominent feature is the ischial tuberosity located on the postero- inferior part of the bone, again making palpation as described in this case impossible. GAS 441; N 243; McM 272)

251 A gynecology resident physician was performing a pelvic examination on a 25-year-old woman who was in the lithotomy position during a physical examination. In order to palpate the patient's uterus, the index and middle fingers of the resident's right hand were inside her vagina while the fingers of his left hand were pressing on the abdomen inferior to the umbilicus. The resident concomitantly lowered his left palm onto the patient's skin and felt a bony structure in the lower midline. Which of the following structures did the resident most likely feel on the palm of his left hand? A. Ilium B. Coccyx C. Sacrum D. Ischium E. Pubis

E(. The linea alba is formed by the intersection of aponeurotic tissues between the right and left rectus abdominal muscles. It contains the aponeuro- ses of the abdominal muscles and is located at the midline of the body. The midaxillary line is oriented vertically in a straight line inferior to the shoulder joint and axilla. The arcuate line (of Douglas) is a curved horizontal line that represents the lower edge of the posterior tendinous portion of the rectus abdominis sheath. An incision at this line will not separate the rectus abdominis sheaths. The semilunar line is represented by an imaginary vertical line below the nipples and usually parallels the lateral edge of the rectus sheath. The tendinous intersections of the rectus abdominis muscles divide the muscle into sec- tions and are usually not well de ned. An incision along these intersections would not divide the two rectus sheaths (GAS Fig. 4-27). GAS 282; N 245; McM 219)

26 The decision is made by emergency department surgeons to perform an exploratory laparotomy on a 32-year-old female with severe abdominal pain. Where would the incision most likely be made to separate the left and right rectus sheaths? A. Midaxillary line B. Arcuate line C. Semilunar line D. Tendinous intersection E. Linea alba

C(. The lateral plantar nerve innervates the inter- ossei and adductor hallucis. These losses would be obvious when the patient attempts to abduct and adduct the toes. Sensation would be absent over the lateral side of the sole, the fifth and fourth toes, and half of the third toe. The medial plantar nerve pro- vides sensation over the plantar surface of the first and second toes and half of the third toe as well as function of the so-called LAFF muscles: first lumbri- cal, abductor hallucis, flexor hallucis brevis, and flexor digitorum brevis. GAS 657-658; N 520, 524, 514; McM 351)

27 A 46-year-old woman stepped on a broken wine bottle on the sidewalk and the sharp glass entered the posterior part of her foot. The patient was admitted to the hospital, and a physical examination concluded that her lateral plantar nerve had been transected (cut through). Which of the following conditions will most likely be confirmed by further physical examination? A. Loss of sensation over the plantar surface of the third toe B. Paralysis of the abductor hallucis C. Paralysis of the interossei and adductor hallucis D. Flexor hallucis brevis paralysis E. Flexor digitorum brevis paralysis

C(. The tendinous arch of fascia pelvis is a dense band of connective tissue that joins the fascia of the levator ani to the feltlike pubocervical fascia that covers the anterior wall of the vagina. If this fascial band is torn, the ipsilateral side of the vagina falls, carrying with it the bladder and urethra, often leading to urinary incontinence. The tendinous arch of the levator ani is a thickened portion of the fascia of the obturator internus and provides part of the origin of the levator ani muscle, but it plays no direct role in incontinence. The coccygeus muscle supports and raises the pelvic oor but is not directly associated with urinary incontinence. The obturator internus is involved with lateral rotation of the thigh. If the rec- tovaginal septum is torn, the patient can be subject to the occurrence of rectocele or enterocele, as the lower portion of the GI tract prolapses into the poste- rior wall of the vagina (GAS Fig. 5-34).GAS 466, 459; N 356; McM 275)

27 After having given birth to five children, a 41-year-old woman seeks correction of chronic urinary incontinence. While relating her history the patient reveals that she has leakage of urine with increased intraabdominal pressure (stress incontinence). An MRI examination reveals injury to the pelvic floor that has altered the position of the neck of the bladder and the urethra. Which of the following structures has most probably been injured at some time during the multiple deliveries? A. Tendinous arch of levator ani B. Coccygeus C. Tendinous arch of fascia pelvis D. Obturator internus E. Rectovaginal septum

A(. Scarpa's fascia is the thick, membranous layer deep to the Camper's adipose fascia in the ante- rior abdominal wall (subcutaneous). Because of the relatively thick, tough nature of connective tissue that makes up Scarpa's fascia, this layer is typically the site to maintain sutures. Camper's fascia is a fatty layer (subcutaneous) and tends not to hold sutures as well, due to the increased cellular content versus the connective tissue found in Scarpa's layer. Transversa- lis fascia is located deep to the abdominal muscula- ture and associated aponeurosis. Extraperitoneal fascia is the deepest layer, adjacent to the parietal peritoneum of the anterior abdominal wall. The ante- rior wall of the rectus sheath is the layer just deep to Scarpa's fascia and super cial to the rectus abdominis muscle anteriorly. The latter three layers are not con- sidered to be super cial fascia. GAS 280; N 321; McM 223)

27 After a "tummy-tuck" (abdominoplasty) procedure is performed on a 45-year-old man, which of the following layers of the abdominal wall will hold the sutures? A. Scarpa's fascia (membranous layer) B. Camper's fascia (fatty layer) C. Transversalis fascia D. Extraperitoneal tissue E. External abdominal oblique aponeurosis

A(. The common fibular (peroneal) nerve passes around the head of the fibula and gives off deep (L4-5) and superficial fibular (peroneal) nerve (L5, S1-2) branches. The two nerves supply the dorsiflex- ors and evertors of the foot, respectively. In this case, the tibialis anterior and extensor digitorum longus are the only muscles listed that are supplied by either of these nerve branches, and both are innervated by the deep fibular (peroneal) nerve. The fibularis (pero- neus) brevis and longus are innervated by the super- ficial fibular (peroneal) nerve and are evertors of the foot. The tibial nerve supplies each of the other muscles listed. GAS 486, 550, 563, 607; N 508, 514; McM 337)

28 A 22 year old male martial arts competitor was examined by the clinician because of pain and serious disability suffered from a kick to the side of his knee. Physical examination revealed a dark bruise just distal to the head of the fibula. Which of the following muscles will most likely be paralyzed? A. Tibialis anterior and extensor digitorum longus B. Tibialis posterior C. Soleus and gastrocnemius D. Plantaris and popliteus E. Flexor digitorum longus and exor hallucis longus

B (Injury to the superior gluteal nerve results in a characteristic motor loss, with paralysis of the gluteus medius and minimus. In addition to their role in abducting the thigh, the gluteus medius and minimus function to stabilize the pelvis. When the patient is asked to stand on the limb of the injured side, the pelvis descends on the opposite side, indicating a positive Trendelenburg test. The gluteal, or lurching, gait that results from this injury is characterized by the pelvis drooping to the unaffected side when the opposite leg is raised. In stepping forward, the affected individual leans over the injured side when lifting the good limb off the ground. The uninjured limb is then swung forward. The gluteus maximus, supplied by the inferior gluteal nerve, is the main muscle respon- sible for allowing a person to rise to a standing posi- tion (extending the exed hip). Spinal nerve roots L1 and L2 and the femoral nerve are responsible for hip exion. Injury to the left superior gluteal nerve would result in sagging of the right side of the pelvis when the affected individual stands on the left limb. The hamstring muscles, mainly responsible for exing the knees to allow a person to sit down from a standing position, are innervated by the tibial branch of the sciatic nerve. GAS 454, 492, 564, 579; N 489; McM 317)

3 A 30-year-old man suffered a superior gluteal nerve injury in a motorcycle crash in which his right lower limb was caught beneath the bike. He is stabilized in the emergency department. Later he is examined and he exhibits a waddling gait and a positive Trendelenburg sign. Which of the following would be the most likely physical nding in this patient? A. Difficulty in standing from a sitting position B. The left side of the pelvis droops or sags when he attempts to stand with his weight supported just by the right lower limb C. The right side of the pelvis droops or sags when he attempts to stand with his weight supported just by the left lower limb D. Weakened exion of the right hip E. Difficulty in sitting from a standing position

A(. The Achilles tendon inserts upon the calca- neus bone. This tendon represents a combination of the tendons of gastrocnemius and soleus muscles. The tendon of the plantaris can insert with this tendon. GAS 621; N 514; McM 345)

30 In an accident during cleanup of an old residential area of the city, the Achilles tendon of a 32-year-old worker was cut through by the blade of a brush cutter. The patient is admitted to the hospital and a laceration of the Achilles tendon is diagnosed. Which of the fol- lowing bones serves as an insertion for the Achilles tendon? A. Calcaneus B. Fibula C. Cuboid D. Talus E. Navicular

C(. In an inversion injury the most common liga- ment involvement comes from the anterior talofibular and calcaneofibular ligaments. The medial plantar nerve is medially located within the sole of the foot and might be injured by traction in an eversion injury, not an inversion injury. The posterior talofibular ligament is located posteriorly and is not usually injured in an inversion injury. The deltoid ligament is located medially and would be injured with an ever- sion injury; it is so strong, however, that eversion is more likely to fracture the medial malleolus rather than tear the deltoid ligament. GAS 638; N 514; McM 349)

31 A 27-year-old female tennis pro injured her ankle during the quarterfinal match. A physical examination at the outpatient clinic revealed a severe inversion sprain of the ankle. Which of the following structures is most commonly damaged in such injuries? A. Medial plantar nerve B. Tibial nerve C. Anterior talofibular ligament D. Posterior talofibular ligament E. Deltoid ligament

A(. Of the answer choices listed, the pubococ- cygeus is the muscle that is most directly associated with the arcus tendineus fascia pelvis and connective tissues of the vagina and the support of the bladder. The obturator internus, piriformis, and coccygeus do not form parts of the levator ani and provide no direct support to the urogenital organs, nor do they have any role in urinary incontinence. The iliococcygeus does form part of the levator ani, but it is located lateral to the pubococcygeus and therefore does not play a direct role in maintaining urinary continence. GAS 454-456; N 336; McM 264)

32 A 34-year-old woman is admitted to the hospital complaining of urinary incontinence. MRI examination reveals that one of the skeletal muscles of the pelvis has a significant tear. Which of the following muscles is the most signi cant in terms of maintaining continence? A. Pubococcygeus B. Obturator internus C. Piriformis D. Coccygeus E. Iliococcygeus

A(. The plantar calcaneonavicular (spring) liga- ment supports the head of the talus and maintains the longitudinal arch of the foot. A fracture of the cuboid bone would not disrupt the longitudinal arch of the foot. Interruption of the plantar aponeurosis is not the best answer because this aponeurosis provides only passive support, unlike the spring ligament. A sprain of the anterior talofibular ligament would result from an inversion injury of the ankle and would not disrupt the longitudinal arch of the foot. A sprain of the deltoid ligament results from eversion of the ankle joint and would not disrupt the longitudinal arch of the foot. GAS 638; N 514; McM 349)

32 A 41-year-old man is admitted to the emergency department with a swollen and painful foot. Radiologic examination reveals that the head of the talus has become displaced inferiorly, thereby causing the medial longitudinal arch of the foot to fall. What would be the most likely cause in this case? A. Tearing of the plantar calcaneonavicular (spring) ligament B. Fracture of the cuboid bone C. Interruption of the plantar aponeurosis D. Sprain of the anterior talofibular ligament E. Sprain of the deltoid ligament

B(. This type of injury can result in the "unhappy triad" (of O'Donoghue) injury, with damage to the medial collateral ligament (MCL), anterior cruciate ligament (ACL), and medial meniscus. A blow to the lateral side of the knee stretches and tears the MCL, which is attached to the medial meniscus. The ACL is tensed during knee extension and can tear subse- quent to the rupture of the MCL. The remaining answer choices describe structures on the lateral surface of the knee, which are not usually injured by this type of trauma. GAS 673; N 496; McM 332)

33 During a football game a 21-year-old wide receiver was illegally blocked by a linebacker, who threw himself against the posterolateral aspect of the runner's left knee. As he lay on the ground, the wide receiver grasped his knee in obvious pain. Which of the following struc- tures is frequently subject to injury from this type of force against the knee? A. Fibular collateral ligament B. Anterior cruciate ligament C. Lateral meniscus and posterior cruciate ligament D. Fibular collateral and posterior cruciate ligament E. All the ligaments of the knee will be affected

E(. The vastus lateralis muscle is located on the lateral aspect of the thigh. The distal portion of this muscle lies superficial to the proximal part of the lateral aspect of the joint capsule of the knee. When a needle is inserted superiorly and laterally to the patella, it penetrates the vastus lateralis muscle on its course to the internal capsule. The short head of biceps femoris has its origin on the posterior aspect of the femur, merges with the long head of the biceps femoris, and inserts on the head of the fibula. The rectus femoris passes longitudinally on the medial aspect of the femur and inserts on the tibial tuberos- ity, via the patellar tendon, or quadriceps tendon. A needle inserted laterally to the patella would not pen- etrate this muscle. The sartorius originates on the anterior superior iliac spine and forms part of the pes anserinus, which inserts on the medial aspect of the proximal part of the tibia. A needle inserted laterally to the patella would not penetrate this muscle. GAS 590-593; N 487; McM 320)

35 A 43-year-old man visits the outpatient clinic with a painful, swollen knee joint. The patient's history reveals chronic gonococcal arthritis. A knee aspiration is ordered for bacterial culture of the synovial uid. A standard suprapatellar approach is used, and the needle passes from the lateral aspect of the thigh into the region immediately proximal to and deep to the patella. Through which of the following muscles would the needle pass? A. Adductor magnus B. Short head of biceps femoris C. Rectus femoris D. Sartorius E. Vastus lateralis

C(. When the popliteus contracts, it rotates the distal portion of the femur in a lateral direction. It also draws the lateral meniscus posteriorly, thereby pro- tecting this cartilage as the distal femoral condyle glides and rolls backward, as the knee is flexed. This allows the knee to flex and therefore serves in unlock- ing the knee. The biceps femoris is a strong flexor of the leg and laterally rotates the knee when it is in a position of flexion. The gastrocnemius is a powerful plantar flexor of the foot. The semimembranosus, similar to the biceps femoris, is a component of the hamstring muscles and is involved in extending the thigh and flexing the leg at the knee joint. The rectus femoris is the strongest quadriceps muscle in extend- ing the leg at the knee. GAS 623-624; N 497; McM 332)

36 A 34-year-old male power lifter visits the outpa- tient clinic because he has difficulty walking. During physical examination it is observed that the patient has a problem unlocking the knee joint to permit flexion of the leg. Which of the following muscles is most likely damaged? A. Biceps femoris B. Gastrocnemius C. Popliteus D. Semimembranosus E. Rectus femoris

B(. The contents of the spermatic cord include ductus deferens; testicular, cremasteric, and deferen- tial arteries; the pampiniform plexus of testicular veins; the genital branch of the genitofemoral nerve; the cremasteric nerves; and the testicular sympathetic plexus and also lymph vessels. The cremaster muscle and fascia originate from the internal abdominal oblique muscle. The external spermatic fascia is derived from the aponeurosis and fascia of the exter- nal oblique muscle. The tunica vaginalis is a con- tinuation of the processus vaginalis (from parietal peritoneum) that covers the anterior and lateral sides of the testes and epididymis. The internal spermatic fascia is derived from the transversalis fascia. The dartos tunic consists of a blending of the adipose (Camper's) and membranous (Scarpa's) layers of the super cial fascia, with interspersed smooth muscle bers. GAS 279; N 365; McM 265)

37 A 35-year-old man is admitted to the hospital with an indirect inguinal hernia. During an open hernioplasty (in contrast to a laparoscopic procedure), the spermatic cord and the internal abdominal oblique muscles are identi ed. Which component of the spermatic cord is derived from the internal abdominal oblique muscle? A. External spermatic fascia B. Cremaster muscle C. Tunica vaginalis D. Internal spermatic fascia E. Dartos fascia

A(. Meromelia is the partial absence of a limb (amelia is total absence). Syndactyly refers to fused digits, and polydactyly is an excess in the number of digits. Central digit ray formation is the underlying mechanism for syndactyly, and talipes equinovarus is a malrotation of the foot, more commonly referred to as clubfoot. 245, 304)

39 Ultrasound of a 20-year-old pregnant woman in the 20th week of pregnancy revealed abnormal limb development of her fetus, showing one arm to be shorter than the other. Which of the following most accurately describes this condition? A. Meromelia B. Central digit ray deformity C. Talipes equinovarus D. Polydactyly E. Syndactyly

E(. The deep fibular (peroneal) nerve is responsi- ble for innervating the muscles of the anterior com- partment of the leg, which are responsible for toe extension, foot dorsiflexion, and inversion. Injury to this nerve will result in foot drop and also loss of sensation between the first and second toes. Injury to the tibial nerve affects the posterior compartment muscles of the leg, which are responsible for plantar flexion and toe flexion, as well as the intrinsic muscles of the sole of the foot. The common fibular (peroneal) nerve splits into the superficial and deep fibular (pero- neal) nerves, and these supply both the lateral and anterior compartments. The superficial fibular (pero- neal) nerve innervates the fibularis (peroneus) longus and brevis muscles, which provide eversion of the foot. If the common fibular (peroneal) nerve were injured, eversion of the foot and plantar flexion would be lost in addition to dorsiflexion and inversion. The saphenous nerve, a continuation of the femoral nerve, is a cutaneous nerve that supplies the medial side of the leg and foot and provides no motor innervation. GAS 627, 660; N 529; McM 337)

4 A 45-year-old man is treated at the hospital after he fell from his bicycle. Radiologic examination reveals fractures both of the tibia and the fibula. On physical examination the patient has a foot drop, but normal eversion (Fig. 5-1). Which of the following nerves is most likely injured? A. Tibial B. Common fibular (peroneal) C. Superficial fibular (peroneal) D. Saphenous E. Deep fibular (peroneal)

C(. Entrapment compression of all or part of the sciatic nerve by the piriformis can mimic disc hernia- tion, most commonly resembling compression of spinal nerve S1. This results in pain down the poste- rior aspect of the thigh and leg and the lateral side of the foot. In this case, loss of sensation over the dorsum of the foot and weakness of foot extension, in addition to eversion, indicate that more than S1 is involved. Foot drop would be anticipated with fibular (pero- neal) nerve involvement. As noted also in a previous question, compression of the common fibular (pero- neal) division of the sciatic nerve by the piriformis gives rise to the clinical condition known as piriformis entrapment. This condition is associated with point pain in the gluteal area, pain in the posterior part of the limb, and possible weakness of muscles in the lateral and anterior compartments of the leg. It can be confused with herniated disc (L5) compression of S1 and sciatica. Paralysis of plantar flexion occurs with a lesion of the tibial division of the sciatic nerve or the tibial nerve. Paralysis of the quadriceps is asso- ciated with pathology of the femoral nerve. Clonic contraction of the adductors could result from obtura- tor nerve problems. GAS 118; N 482, 490; McM 317)

40 A 43-year-old woman is examined by a neurolo- gist, to whom she complains of pain in her lower limb of 6 months' duration. She has pain in the gluteal area, thigh, and leg. The neurologist observes reduced sensa- tion over the dorsum and lateral side of the involved foot and some weakness in foot dorsiflexion and ever- sion. A diagnosis of a piriformis entrapment syndrome is made, with compression of the fibular (peroneal) division of the sciatic nerve. Which of the following conditions did the neurologist also most likely find during her physical examination of the patient? A. Paralysis of plantar flexion B. Instability of the knee, due to paralysis of the quadriceps femoris C. Foot drop D. Spasm or clonic contractures of the adductor musculature of the thigh E. Loss of sensation in the gluteal area, by paralysis of anterior cluneal nerves

E(. The AER secretes growth factor, which initi- ates outgrowth of the limb mesenchyme that initiates formation of the limb bud. 235, 239)

40 A male infant is born with cleft hand, also known as "lobster-claw hand." This abnormality is caused by the apical ectodermal ridge (AER) failing to properly develop. Which of the following best describes the principal function of the AER? A. Establishes the anteroposterior axis of the limb bud B. Stimulates blood vessel growth into the limb bud C. Stimulates cartilage differentiation in the limb bud D. Stimulates nerve growth into the limb bud E. Stimulates outgrowth of the limb bud

C(. The arcuate line is a horizontal line that demarcates the lower limit of the posterior aponeu- rotic portion of the rectus sheath. It is also where the inferior epigastric vessels perforate the sheath to enter the rectus abdominis. The intercristal line is an imaginary line drawn in the horizontal plane at the upper margin of the iliac crests. The linea alba is a tendinous, median raphe running vertically between the two rectus abdominis muscles from the xiphoid process to the pubic symphysis. The pectineal line is a feature of the superior ramus of the pubic bone; it provides an origin for the pectineus muscle of the thigh and medial insertions for the abdominal obliques and transversus muscles. The semilunar line is the curved, vertical line along the lateral border of the sheath of the rectus abdominis. GAS 287; N 247; McM 220)

40 After a mastectomy, a musculocutaneous flap is used to restore the thoracic contour in a 34-year-old female patient. The ipsilateral (same side) rectus abdominis muscle was detached carefully from the surrounding structures and transposed to the thoracic wall. Which of the following landmarks is most often used to locate the inferior end of the posterior, tendinous layer of the rectus sheath? A. Intercristal line B. Linea alba C. Arcuate line D. Pectineal line E. Semilunar line

D(. Polydactyly describes the presence of super- numerary digits of the hands and feet. Cleft foot occurs when one or more digital rays fail to develop, causing absence of the central digits. Clubfoot is a malrotation of the foot around the axis, and amelia is complete absence of a limb. Syndactyly is absence of digits either due to failure of digital rays to form or incomplete apoptosis.)

41 A 42-year-old woman gave birth to an 8 lb baby girl with two additional toes to the right of her left big toe. Which of the following best describes this limb anomaly? A. Amelia B. Cleft foot C. Club foot D. Polydactyly E. Syndactyly

D(. The psoas muscles (covered in psoas fascia) originate from the transverse processes, intervertebral disks, and bodies of the vertebral column at levels T12 to L5. In the image, this fascia contains a calci ed tuberculous abscess. The pancreas is an elongated organ located across the back of the abdomen, behind the stomach. The tapering body extends horizontally and slightly upward to the left and ends near the spleen. The cecum is the blind-ending pouch of the ascending colon, lying in the right iliac fossa. The fundus of the stomach lies inferior to the apex of the heart at the level of the fth rib. The suspensory liga- ment of the duodenum is a bromuscular band that attaches to the right crus of the diaphragm. GAS 371; N 258; McM 257)

41 An anteroposterior radiograph is taken of the lumbar region in a 31-year-old female patient who had been treated for tuberculous spondylitis at vertebral levels T12 to L1. The patient has been asymptomatic for 10 years. A density is detected; a calcified tuberculous abscess is suspected. Which of the following is the most likely site of the calcified tuberculous abscess? A. Body of pancreas B. Cecum C. Fundus of stomach D. Psoas fascia E. Suspensory ligament of the duodenum

A(. The superior gluteal nerve innervates the gluteus medius, gluteus minimus, and tensor fasciae latae muscles. The tensor fasciae latae arises from the iliac crest, inserts into the iliotibial tract of the lateral aspect of the thigh, and assists in exion of the hip. The rectus femoris is innervated by the femoral nerve; it exes the hip and extends the knee, thus acting upon two major joints. It arises in part from the ante- rior inferior iliac spine and the rim of the acetabulum and inserts into the quadriceps tendon. The gluteus maximus is supplied by the inferior gluteal nerve. The piriformis and quadratus femoris are both short lateral rotators of the hip and are supplied by branches of the sacral plexus. GAS 565, 575; N 490; McM 320)

41 Three years following a 62-year-old's hip replace- ment, the man's CT scans indicated that two of his larger hip muscles had been replaced by adipose tissue. The opinion is offered that his superior gluteal nerve could have been injured during the replacement proce- dure, and the muscles supplied by that nerve had atro- phied and been replaced by fat. Which of the following muscles receives its innervation from the superior gluteal nerve? A. Tensor fasciae latae B. Rectus femoris C. Gluteus maximus D. Piriformis E. Quadratus femoris

A(. The ischial spine is the correct bony land- mark used to administer a pudendal nerve block. The pudendal nerve crosses the sacrospinous ligament, which attaches to the ischial spine. Accessing the ischial spine and thus the pudendal nerve is done most easily using a transvaginal approach. The pos- terior superior and inferior iliac spines are located on the posterior aspect of the pelvis and articulate with the lateral aspect of the sacrum. They do not relate to the course of the pudendal nerve. The ischial tuberosi- ties are the most inferior aspect of the bony pelvis. The skin and soft tissues around the ischial tuberosi- ties receive sensory supply from the pudendal nerve and perineal branches of the posterior femoral cutane- ous nerve. Injections into the area around the tuber- osities are less certain, however, than injections at the sacrospinous ligament and often fail to anesthetize the anal triangle well. The coccyx is a poor target for locating and anesthetizing the pudendal nerve. GAS 441-445; N 391; McM 92)

43 A 34-year-old pregnant woman is prepared in the hospital for delivery. The gynecologist decides to perform a pudendal nerve block using a transvaginal approach. Which bony structure would be the most reliable as a landmark to block the pudendal nerve? A. Ischial spine B. Posterior inferior iliac spine C. Ischial tuberosity D. Posterior superior iliac spine E. Coccyx

B(. Contraction of the gastrocnemius on the frac- tured calcaneus would increase the pain because the gastrocnemius inserts with the soleus upon that bone, via the calcaneal tendon, or tendo Achilles. The flexor digitorum profundus passes the ankle medially to enter the sole of the foot, where it inserts upon the distal phalanges. The tibialis posterior, likewise, passes under the medial malleolus, with complex insertions upon the navicular bone, cuneiform bones, metatarsal bones, and the cuboid bone. The tibialis anterior, a muscle of the anterior leg compartment, inserts upon the navicular bone and, with the tibialis posterior, is a strong invertor of the foot. The fibularis (peroneus) longus is a muscle of the lateral compartment of the leg. It passes under the lateral malleolus, entering the sole of the foot by crossing the lateral surface of the calcaneus, and inserts primarily into the medial cuneiform and base of the first metatarsal bone. GAS 621, 636; N 503; McM 339)

43 A 49-year-old male worker fell from a ladder, with his weight impacting on the heels of his feet. Radiologic examination reveals comminuted calcaneal fractures. After the injury the contraction of which one of the following muscles could most likely increase the pain in the injured foot? A. Flexor digitorum profundus B. Gastrocnemius C. Tibialis posterior D. Tibialis anterior E. Fibularis (peroneus) longus

C(. A severe injury of the tibial nerve in the pop- liteal fossa would result in a dorsiflexed and everted foot because of the intact muscles of the extensor (anterior) and evertor (lateral) compartments of the leg. It would result also in some weakening of knee flexion because of loss of the gastrocnemius muscle, which flexes the knee and plantar flexes the foot. The hamstrings also flex the knee, so this function would not be lost. Plantar flexion at the ankle would be paralyzed with the loss of the gastrocnemius and soleus, in addition to the flexors of the toes, and inver- sion by the tibialis posterior. Foot drop results from loss of the anterior compartment, innervated by the deep fibular (peroneal) nerve. GAS 545; N 514; McM 345)

44 A 24-year-old woman received a small-caliber bullet wound to the popliteal fossa from a drive-by assailant. The patient was admitted to the emergency department, where the surgeons recognized that the bullet had severed the tibial nerve. Such an injury would most likely result in which of the following? A. Inability to extend the leg at the knee B. Foot drop C. A dorsiflexed and everted foot D. A plantar flexed and inverted foot E. Total inability to flex the leg at the knee joint

B(. The ilioinguinal nerve, which arises from the L1 spinal nerve, innervates the skin on the medial aspect of the thigh, scrotum (or labia majora), and the mons pubis. It has been injured in this patient. The genitofemoral nerve splits into two branches: The genital branch supplies the scrotum (or labia majora) whereas the femoral branch supplies the skin of the femoral triangle. The subcostal nerve has a lateral cutaneous branch that innervates skin in the upper gluteal region, in addition to distribution over the lower part of the anterior abdominal wall. The iliohy- pogastric nerve innervates the skin over the iliac crest and the hypogastric region. Spinal nerve T9 supplies sensory innervation to the dermatome at the level of T9, above the level of the umbilicus. GAS 398; N 253; McM 220)

44 A 32-year-old woman was admitted to the hospital with a complaint of pain over her umbilicus. Radiographic examination revealed acute appendicitis. The appendix was removed successfully in an emergency appendectomy. One week postoperatively the patient complained of paresthesia of the skin over the pubic region and the anterior portion of her perineum. Which of the following nerves was most likely injured during the appendectomy? A. Genitofemoral B. Ilioinguinal C. Subcostal D. Iliohypogastric E. Spinal nerve T9

C(. The obturator artery provides the artery within the ligament of the head of the femur (in about 60% of cases), the artery that supplies the head of the femur, primarily during childhood, later becoming atretic. In the adult this artery supplies only the area of the fovea of the head of the femur. The ligament of the head of the femur arises from the acetabular notch, thereafter receiving the little artery. In some individuals the medial circum ex femoral gives origin to the artery of the head. In the adult the arterial supply of the neck and head is provided by intracap- sular branches of the medial circum ex femoral and lateral circum ex femoral arteries that pierce the neck of the femur, with some supply also from the gluteal arteries. The lateral circum ex femoral artery arises from the deep femoral and supplies the vastus latera- lis. The pudendal artery arises from the internal iliac and provides blood supply for the structures of theperineum. Quite often, when an older patient with osteoporosis has a hip fracture, the femoral neck may have fractured, precipitating a fall, rather than the fall resulting in the hip fracture. GAS 558, 568, 676; N 491; McM 326)

45 An 82-year-old grandmother slipped on the pollished floor in her front hall and was transported to the emergency department and admitted for examination with a complaint of great pain in her right lower limb. During physical examination it is observed by the resident that the right lower limb is laterally rotated and noticeably shorter than her left limb. Radiologic examination reveals an intracapsular fracture of the femoral neck. Which of the following arteries supplies the head of the femur in early childhood but no longer in a patient of this age? A. Superior gluteal B. Lateral circumflex femoral C. A branch of the obturator artery D. Inferior gluteal E. Internal pudendal

A(. With sufficient downward force, the head of the talus can break through the plantar calcaneona- vicular (spring) ligament, causing the medial longitu- dinal arch of the foot to fall, forcing the anterior part of the foot into abduction. The plantar calcaneona- vicular ligament is attached between the sustentacu- lum tali of the calcaneus and the medial surface of the navicular bone, with the head of the talus lying directly upon the inner surface of the ligament. The cuboid bone is located lateral and anterior to the talus bone and would not be fractured. The plantar aponeurosis, a dense, wide band of tissue beneath the fascia of the sole, attaches to the calcaneus and ends distally in longitudinal bands to each of the toes. It stretches very little, even under very heavy loads, and would not rupture in this case. The ante- rior talofibular ligament is very often injured in "sprained ankle" but would not be directly involved here. The distal tibiofibular joint is a fibrous (and usually nonsynovial) type of joint (called a syndes- mosis) between the tibia and fibula, not involved in the displacement of the talus bone (GAS Fig. 6-98). GAS 539-542, 648; N 514; McM 349)

46 A 19-year-old patient is admitted to the orthope- dic service with a complaint of severe pain in his very swollen and discolored foot. He states that he hurt the foot when jumping from his girlfriend's bedroom window to the concrete driveway below. Plain film radiographic studies reveal that the head of the talus has become displaced inferiorly, thereby causing the medial longitudinal arch of the foot to fall. What would be the most likely, serious problem in such a case? A. Tearing of the plantar calcaneonavicular (spring) ligament B. Fracture of the cuboid bone C. Interruption of the plantar aponeurosis D. Sprain of the anterior talofibular ligament E. Disruption of the distal tibiofibular ligament

E(. The lateral femoral cutaneous nerve leaves the pelvis laterally, about 2 cm medial to the anterior superior iliac spine, passing beneath, or through, the inguinal ligament. As a consequence of its site of exit, any tension upon or compression of the inguinal liga- ment can affect the nerve. If it is thus affected, the individual may feel burning sensations or pain along the lateral aspect of the thigh, which is the region of distribution of the nerve. Obesity, sudden weight loss, wearing a heavy gun belt, wearing trousers that are too tight (Calvin Klein syndrome), or having someone sitting on another's lap for an extended period of time can lead to meralgia paresthetica, the painful lateral thigh. The femoral nerve emerges from beneath the middle of the inguinal ligament and is not usually affected by similar traction or compression. The obtu- rator nerve leaves the pelvis through the obturator canal and enters the thigh deeply in a protected loca- tion. It innervates the adductor muscles and supplies sensation on the medial aspect of the thigh. The fibular (peroneal) division of the sciatic nerve sup- plies the muscles of the anterior and lateral compart- ments of the leg and provides sensory fibers for the dorsum and lateral side of the foot. The superior gluteal nerve provides motor supply to the gluteus medius and minimus muscles. GAS 486; N 525; McM 322)

47 A 29-year-old male police officer is examined in a neighborhood clinic, with a complaint of discomfort in the lateral thigh. The physician observes that the policeman is rather overweight and that he is wearing a heavy leather belt, to which numerous objects are attached, including his empty holster. After a thorough physical examination a diagnosis of meralgia paresthetica is confirmed. Which of the following nerves is most likely involved? A. Superior gluteal B. Femoral C. Obturator D. Fibular (peroneal) division of sciatic E. Lateral femoral cutaneous

D(. The tibialis posterior tendon is the most ante- rior of the structures that pass under the laciniate ligament (flexor retinaculum) on the medial side of the ankle to enter the sole of the foot. Increases of pressure within the tissues of the plantar aspect of the foot, usually due to increased fluid from hemorrhage, inflammatory processes, or infections, cause tarsal tunnel syndrome, comparable to carpal tunnel syn- drome of the hand. The plantar aponeurosis and other fibrous and osseous tissues of the plantar surface cause this area to be relatively nondistensible; there- fore, it takes little increase of fluid content to result in pressures adequate to restrict venous drainage and, thereafter, arterial inflow to the region. Fasciotomy of the medial skin and fascia of the foot and the posterior compartment of the leg can be required to reduce the pressure and allow healing to take place. The struc- tures that pass beneath the flexor retinaculum are, from anterior to posterior: Tendon of tibialis poste- rior; tendon of flexor Digitorum longus; posterior tibial Vessels and Nerve; tendon of flexor Hallucis longus. (This is the basis of the mnemonic: "Tom, Dick, and a Very Nervous Harry.") Neither the plan- taris tendon nor the tibialis anterior tendon pass through this canal (GAS Figs. 6-105 and 6-110). GAS 621-623; N 514; McM 346)

48 The swollen and painful left foot of a 23-year-old female long distance runner is examined in the univer- sity orthopedic clinic. She states that she stepped on an unseen sharp object while running through the park several days earlier. Emergency surgery is ordered to deal with her tarsal tunnel syndrome. The tarsal tunnel is occupied normally by tendons, vessels, and nerves that pass beneath a very strong band of tissue (flexor retinaculum) on the medial side of the ankle. What is the most anterior of the structures that pass through this tunnel? A. Flexor hallucis longus tendon B. Plantaris tendon C. Tibialis anterior tendon D. Tibialis posterior tendon E. Tibial nerve

A(. The key distinguishing feature of a direct inguinal hernia is that it does not pass through the deep inguinal ring; it passes through the lower portion of the inguinal triangle (of Hesselbach). This triangle is bordered laterally by the inferior epigastric artery and vein; medially, it is bordered by the lateral edge of rectus abdominis; inferiorly, it is bordered by the iliopubic tract and inguinal ligament. An indirect hernia passes through the deep inguinal ring and into the inguinal canal. It often descends through the super cial ring into the scrotum or labium, a feature less common in a direct inguinal hernia. If the tip of the examiner's little nger is inserted into the super- cial ring and the patient is asked to cough, an indi- rect inguinal hernia may be felt hitting the very tip of the examining nger. A direct inguinal hernia will be felt against the side of the digit. Both types of inguinal hernias occur above the inguinal ligament, and both are present lateral to the lateral border of the rectus abdominis. The pubic symphysis, a midline joint between the two pubic bones, provides no informa- tion for distinguishing types of hernias. The femoral canal, a feature of the femoral sheath, passes beneath the inguinal ligament into the thigh, providing the pathway taken by a femoral hernia. The pectineal ligament lies behind, or deep to, the proximal end of the femoral canal (GAS Fig. 4-50A). GAS 299-301; N 257; McM 220)

49 A 45-year-old man is admitted to the hospital with a massive hernia that passes through the inguinal triangle (of Hesselbach). Which of the following structures is used as a landmark to distinguish a direct inguinal hernia from an indirect inguinal hernia? A. Inferior epigastric vessels B. Femoral canal C. Inguinal ligament D. Rectus abdominis muscle (lateral border) E. Pectineal ligament

A(. The femoral ring is the abdominal opening of the femoral canal. A femoral hernia passes through the femoral ring into the femoral canal deep and infe- rior to the inguinal ligament. It can appear as a bulging at the saphenous hiatus (fossa ovalis) of the deep fascia of the thigh, the hiatus through which the saphenous vein passes to the femoral vein. The super- cial inguinal ring is the triangular opening in the aponeurosis of the external abdominal oblique and lies lateral to the pubic tubercle. The deep inguinal ring lies in the transversalis fascia lateral to the infe- rior epigastric vessels. Herniation into either of these two openings is associated with an inguinal hernia. The obturator canal, a bony opening between the superior and inferior ramus of the pubic bone, is the site of an obturator hernia. GAS 301, 545, 573; N 487; McM 225)

5 A 49-year-old male construction worker is admitted to the emergency department with a painful lump on the proximal medial aspect of his thigh. Radiologic and physical examinations reveal that the patient has a herniation of abdominal viscera beneath the inguinal ligament into the thigh. Through which of the following openings will a hernia of this type initially pass to extend from the abdomen into the thigh? A. Femoral ring B. Super cial inguinal ring C. Deep inguinal ring D. Fossa ovalis E. Obturator canal

B(. The tibial nerve divides into the medial and lateral plantar nerves on the medial side of the ankle. These two nerves provide sensation for the sole of the foot. Sensory supply to the dorsum of the foot is pro- vided mostly by the superficial fibular (peroneal) nerve, with the deep fibular (peroneal) nerve provid- ing sensation for the skin between the first and second toes. Foot drop would be caused by interruption of the common fibular (peroneal) nerve. Sensory loss to the lateral side of the foot results from loss of the sural nerve. Paralysis of the extensor digitorum brevis would be attributed to injury to the terminal motor branch of the deep fibular (peroneal) nerve. GAS 660; N 528; McM 346)

50 A 22-year-old man is admitted to the emergency department after falling from his bicycle. Radiologic examination reveals a fracture of the tibia above the ankle. MRI and physical examination reveal that the tibial nerve is severed on the posterior aspect of the tibia. Which of the following signs will most likely be present during physical examination? A. Sensory loss of the dorsum of the foot B. Sensory loss on the sole of the foot C. Foot drop D. Paralysis of the extensor digitorum brevis E. Sensory loss of the entire foot

B(. The gastrocnemius muscle arises from the femur just proximal to the femoral condyles. This strong muscle could displace the distal fragment of the fractured femur posteriorly. In addition, the pop- liteal artery is the deepest structure in the popliteal fossa (right against the popliteal surface of the distal femur) and is susceptible to laceration in this scenario as the fractured end of the distal femoral fragment is pulled against the popliteal artery. Orthopedic sur- geons always look for damage to the popliteal artery in a patient with a supracondylar fracture. The soleus arises from the tibia and would have no effect upon the femur. The semitendinosus arises from the ischial tuberosity and inserts medially on the proximal tibia, via the pes anserinus. The tibialis anterior arises from the tibia and inserts mostly onto the navicular bone. GAS 621-623; N 503; McM 301)

51 A 24-year-old man is admitted to the emergency department after a car collision. Radiologic examina- tion reveals a fracture at the junction of the middle and lower thirds of the femur. An MRI examination provides evidence that the popliteal vessels were injured when the distal fragment of the fracture was pulled posteri- orly. Which of the following muscles is most likely to displace the distal fracture fragment? A. Soleus B. Gastrocnemius C. Semitendinosus D. Gracilis E. Tibialis anterior

E(. The posterior tibial artery provides most of the arterial supply for the neck and body of the talus bone. The fibular (peroneal) artery provides a small amount of vascular supply. The medial plantar and lateral plantar branches of the posterior tibial artery are distributed to tissues in the plantar surface of the foot. The dorsalis pedis is the continuation of the anterior tibial artery on the dorsum of the foot. GAS 626, 657; N 509; McM 344)

52 A 65-year-old man is admitted to the hospital after falling from his roof while cleaning leaves and pine needles from the gutters. Among other injuries suffered in his fall, radiologic examination reveals a fracture of the talus bone in one foot. Much of the blood supply of this bone can be lost in such an injury and can result in osteonecrosis. From what artery does this bone receive its primary vascular supply? A. Medial plantar B. Lateral plantar C. Dorsalis pedis D. Anterior tibial E. Posterior tibial

B(. The deep fibular (peroneal) nerve supplies the dorsiflexors of the foot, including the extensor hallucis longus and extensor digitorum longus. It also supplies the tibialis anterior, an invertor of the foot. This nerve has sensory distribution only to the skin between the first two toes. The common fibular (pero- neal) nerve supplies not only the preceding muscles but also the evertors of the foot and provides sensa- tion for most of the dorsum of the foot. The sciatic nerve innervates the muscles of the posterior thigh and all muscles of the leg and foot, in addition to providing sensory supply in those areas. The superfi- cial fibular (peroneal) nerve innervates the evertors of the foot and provides sensation for the dorsum of the foot. The tibial nerve is the nerve for muscles of the posterior compartment of the leg and also of the plantar region and supplies sensation over the medial aspect of the leg posteriorly and the plantar surface of the foot and toes. GAS 626, 660; N 508; McM 337)

54 A 34-year-old male long-distance runner com- plained to the team physician of swelling and pain of his shin. Skin testing in a physical examination showed normal cutaneous sensation of the leg. Muscular strength tests showed marked weakness of dorsiflexion and impaired inversion of the foot. Which nerve serves the muscles involved in the painful swelling? A. Common fibular (peroneal) B. Deep fibular (peroneal) C. Sciatic D. Superficial fibular (peroneal) E. Tibial

B(. The medial plantar nerve innervates the abductor hallucis and both flexor hallucis longus and brevis. This nerve also provides motor supply for the flexor digitorum brevis and the first lumbrical. The lateral plantar nerve innervates all other intrinsic muscles in the plantar region of the foot. The sural nerve is sensory to the lateral posterior leg and lateral side of the foot; it arises from a combination of branches of the tibial nerve and common fibular (peroneal) nerve. The deep fibular (peroneal) nerve supplies dorsiflexors, toe extensors, and invertors of the foot. GAS 660; N 522; McM 351)

55 A 7-year-old girl accidentally stepped on a sharp snail shell while walking to the beach. She was admitted to the hospital, where she received a tetanus shot, and the wound was cleaned thoroughly and sutured. One week later, during a return visit to her physician, it is seen that she has great difficulty in flexing her big toe, even though there is no inflammation present in the sole of the foot. Which nerve was most likely damaged by the piercing of the shell? A. Lateral plantar nerve B. Medial plantar nerve C. Sural nerve D. Superficial fibular (peroneal) nerve E. Deep fibular (peroneal) nerve

B(. Blood ow would be impeded or greatly reduced in the left testicular vein because of the occlu- sion of the left renal vein—into which the left testicu- lar vein drains. This would result in pain as the testicular venous vessels become swollen. The tes- ticular artery originates from the abdominal aorta more inferiorly and is not being compressed. Pain mediated from the renal organs would pass to the T11 and T12 spinal cord levels via the least thoracic splanchnic nerves. There would be no compression of lumbar splanchnic nerves in this case. Compression of the preaortic sympathetics would not produce pain, nor would it cause referral of pain. Visceral afferents for pain terminate at the T7 level of the spinal cord. The vagus, a parasympathetic nerve, does not carry visceral pain bers in the abdomen; pain is mediated by branches of the sympathetic trunks. GAS 516; N 310; McM 258)

56 A 57-year-old man is admitted to the emergency department with pain in his left flank and testicles. Laboratory tests indicate hematuria and anemia. A CT scan examination provides evidence that blood flow in the left renal vein is being occluded where it crosses anterior to the aorta. Which of the following is the most likely cause of the testicular pain? A. Compression of the testicular artery B. Occlusion of flow of blood in the testicular vein C. Compression of the afferent fibers in the lumbar splanchnic nerves D. Compression of the sympathetic bers in the preaortic plexus E. Compression of the posterior vagus nerve

C(. The deep fibular (peroneal) nerve supplies the fibularis (peroneus) tertius muscle. Although its name might lead one to think that this muscle is in the lateral compartment with the other two fibularis (peroneus) muscles, it is in the anterior (extensor) compartment of the leg. It is named for its origin from the fibula. It inserts upon the dorsum of the base of the fifth (or fourth) metatarsal bone and assists in extension and eversion of the foot. The sural nerve is a cutaneous nerve, formed by contribu- tions from the tibial and common fibular (peroneal) nerves; it supplies the posterior lateral leg and the lateral side of the foot. The lateral plantar nerve is a branch of the tibial nerve; it innervates the quadratus plantae, muscles of the little toe, the adductor hallu- cis, lumbricals 2 to 4, and all of the interossei. It is sensory to the lateral side of the sole and the lateral three and a half digits. The superficial fibular (pero- neal) nerve supplies the fibularis (peroneus) longus and brevis and innervates the skin on most of the dorsum of the foot. The tibial nerve supplies the calf muscles and divides into the medial and lateral plantar nerves. GAS 628-631; N 508; McM 347)

57 Young parents were concerned that their 14-month- old daughter had not yet begun walking. Their pediatri- cian reassured them, saying that one of the muscles of the leg, the fibularis (peroneus) tertius, had to complete its central neurologic development before the child could lift the outer corner of the foot and walk without stumbling over her toes. What is the most common nerve supply of this muscle? A. Sural B. Lateral plantar C. Deep fibular (peroneal) D. Superficial fibular (peroneal) E. Tibial

D(. Indirect inguinal hernia is the most common groin hernia in females. Although femoral hernias occur more commonly in females than in males, the occurrence of indirect inguinal hernias in women is greater. Inguinal hernias are much more common in males than in females. Epigastric and umbilical hernias would not present with pain to the inguinal region. Direct inguinal hernias, while exhibiting equal incidence in both sexes, are not the most common female hernia. GAS 299-301; N 257; McM 225 )

58 A 39-year-old woman is admitted to the hospital with pain radiating to her inguinal region. Radiologic and physical examination reveal a herniation. Which of the following is the most common type of hernia in a female patient? A. Femoral hernia B. Umbilical hernia C. Direct inguinal hernia D. Indirect inguinal hernia E. Epigastric hernia

B(. The obturator nerve innervates the adductor muscles, including the gracilis, pectineus, and obtura- tor externus. The tibial nerve supplies the calf muscles and intrinsic muscles in the plantar portion of the foot. The inferior gluteal nerve innervates the gluteus maximus; the superior gluteal nerve supplies the gluteus medius and minimus and tensor fasciae latae. The femoral nerve provides motor supply to the quad- riceps femoris, sartorius, and, in some cases, the pec- tineus. This gait pattern is characteristic of hypertonia in the lower limb. As a result these areas become flexed to various degrees, giving the appearance of crouching, while tight adductors produce extreme adduction. GAS 468, 500, 563-565; N 488; McM 319 )

59 A 45-year-old intoxicated man was struck by a tour bus while walking in the middle of the street. The man was admitted to the emergency department and during physical examination was diagnosed with "adductor gait," in which an individual crosses one limb in front of the other, due to powerful hip adduc- tion. Which of the following nerves was most likely involved in this condition? A. Tibial B. Obturator C. Inferior gluteal D. Superior gluteal E. Femoral

A(. The talocrural (tibiotalar, ankle) joint is a hinge-type synovial joint between the tibia and talus. It permits dorsiflexion and plantar flexion, and frac- ture of this joint would affect these movements. GAS 635-645; N 514; McM 354)

6 A 37-year-old man is admitted to the hospital after an injury to his foot while playing flag football with friends on a Saturday morning. A series of radio- graphs demonstrates a fracture involving the talocrural (tibiotalar, ankle) joint. Which movements are the major ones to be affected by this injury? A. Plantar flexion and dorsiflexion B. Inversion and eversion C. Plantar flexion, dorsiflexion, inversion, and eversion D. Plantar flexion and inversion E. Dorsiflexion and eversion

D(. The transversalis fascial layer is the source of the internal spermatic fascia. The walls of the sper- matic cord consist of three layers: external spermatic fascia, cremaster muscle, and the internal spermatic fascia. The external spermatic fascia is an extension of the external oblique fascia and aponeurosis. The cremaster muscle is a derivative of the internal oblique abdominal muscle and its fascia. The processus vagi- nalis is a pouch of peritoneum that precedes the testis as it descends through the deep inguinal ring and inguinal canal in the seventh month of development. That portion of the processus that is normally retained forms the tunica vaginalis of the testis. Retention of the proximal part of the processus provides a pathway for a congenital indirect inguinal hernia. If a portion of the intermediate part of the processus remains, it can form a uid- lled hydrocele. GAS 296-297; N 257; McM 220 )

60 A 23-year-old man is admitted to the hospital with a bulge in his scrotum. Physical examination reveals an indirect inguinal hernia. During the open hernia repair the internal spermatic fascia is identified and re ected to expose the ductus deferens and testicular vessels. From which of the following does the internal spermatic fascial layer of the spermatic cord develop? A. External abdominal oblique aponeurosis B. Internal abdominal oblique aponeurosis C. Transversus abdominis aponeurosis D. Transversalis fascia E. Processus vaginalis

D(. The dorsalis pedis is the continuation of the anterior tibial artery into the foot, as it passes the distal end of the tibia and the ankle joint. The pulse of the dorsalis pedis can be felt between the tendon of the extensor hallucis longus and the tendon of the extensor digitorum longus to the second toe. A strong pulse is a positive indicator of circulation through the limb. The fibular (peroneal) artery is a branch of the posterior tibial artery and passes in the calf between the flexor hallucis longus and tibialis posterior, making it difficult to palpate. The deep plantar artery, the extension of the first dorsal interosseous or lateral plantar arteries, passes deep to the aponeurotic tissues and central muscles of the foot, making palpation unlikely. The dorsal metatarsal branches of the dorsa- lis pedis pass under cover of the extensor digitorum longus and brevis tendons. Palpable pulses of the first or other dorsal metatarsal arteries can therefore be difficult to detect (GAS Fig. 6-119). GAS 658; N 518; McM 344)

61 A 55-year-old man is admitted to the hospital for an iliofemoral bypass. The operation is performed suc- cessfully and the blood flow between the iliac and femoral arteries is restored. During rehabilitation which of the following arteries should be palpated to monitor good circulation of the lower limb? A. Anterior tibial B. Deep fibular (peroneal) C. Deep plantar D. Dorsalis pedis E. Dorsal metatarsal

D(. Injury to the dorsalis pedis artery on the dorsum of the foot can also cause trauma to the ter- minal portion of the deep fibular (peroneal) nerve. In the proximal part of the foot, this could result in loss of sensation between the first and second toes and paralysis of the extensor digitorum brevis and the extensor hallucis brevis muscles. In the distal part of the foot, only the sensory loss might be apparent. Clubfoot is a congenital malformation observed in pediatric patients. This syndrome combines plantar flexion, inversion, and adduction of the foot. Neither extension of the big toe by the extensor hallucislongus nor paralysis of the tibialis anterior (weakness of foot inversion) would occur by this injury because both of these muscles are innervated by the deep fibular (peroneal) nerve much more proximally in the leg.)

62 A 55-year-old woman is bitten by a dog in the dorsum of the foot and is admitted to the emergency department. The wound is cleaned thoroughly, during which it is seen that no tendons have been cut, but the dorsalis pedis artery and the accompanying nerve have been injured. Which of the following conditions would be expected during physical examination? A. Clubfoot B. Foot drop C. Inability to extend the big toe D. Numbness between the first and second toes E. Weakness in inversion of the foot

D(. The sural nerve is formed by contributions from the tibial nerve and a branch from the common fibular (peroneal) nerve. It provides sensation for the lower lateral portion of the calf and continues beneath the lateral malleolus as the lateral cutaneous nerve of the foot. It is often used for nerve grafting procedures as well as biopsied for diagnostic purposes. When it is grafted to the "living end" of a cut motor or sensory nerve, the severed nerve processes within the "living" nerve grow into the sural nerve sheath, using it as a guide to the distal, surgically anastomosed nerve. Thus, axons from a branch of a functional motor nerve can grow to reinnervate paralyzed muscles. In this case, the surgeon would connect portions of the sural nerve to the functional facial nerve, tunnel it to the opposite side of the face, and join it surgically to the branches of the paralyzed nerve, where it would grow through the now empty nerve sheaths (due to Wallerian degeneration) to the muscles. Growth and reinnervation usually occur at a rate of 1 mm/day (or 1 inch/month) so the time estimated before reinnerva- tion is based on the distance the regenerating fibers need to traverse. The tibial nerve supplies muscles and sensation to the calf and plantar surface of the foot. The common fibular (peroneal) nerve innervates the lateral and anterior compartment muscles and sensation to the dorsum of the foot. The saphenous nerve accompanies the great saphenous vein on the medial side of the leg and foot. GAS 630, 662; N 529; McM 340)

63 A 31-year-old woman presents to the department of surgery with a complaint of facial paralysis (Bell's palsy), which had appeared a year earlier and had resulted in paralysis of muscles of one side of her face. The chief of plastic surgery recommends a nerve graft, taking a cutaneous nerve from the lower limb to replace the defective facial nerve. The surgery is successful. Six months after the procedure, there is restoration of func- tion of previously paralyzed facial muscles. There is an area of skin on the back of the leg laterally and also on the lateral side of the foot that has no sensation. What nerve was used in the grafting procedure? A. Superficial fibular (peroneal) B. Tibial C. Common fibular (peroneal) D. Sural E. Saphenous

C(. The anterior cutaneous branch of the iliohy- pogastric nerve is responsible for the innervation of the skin above the mons pubis. This nerve arises from the T12 and L1 spinal nerves and runs transversely around the abdominal wall and over the lowest portion of the rectus sheath. It is the rst cutaneous nerve situated superior to the mons pubis. Nerves from the T11 and the T12 ventral rami terminate below the umbilicus but superior to the mons pubis. The ilioin- guinal nerve courses through the inguinal canal, com- monly on the lateral side of the spermatic cord and is therefore typically inferior to the incision. The lateral femoral cutaneous nerve travels lateral to the psoas muscle and emerges from the abdomen about an inch medial to the anterior superior iliac spine, passing thereafter to the lateral aspect of the thigh. GAS 398; N 253; McM 253 )

63 A 36-year-old woman is admitted to the hospital for the imminent birth of her baby. The decision is made to perform an emergency cesarean section. A Pfannenstiel incision is used to reach the uterus by making a transverse incision through the external sheath of the rectus muscles, about 2 cm above the pubic bones. It follows natural folds of the skin and curves superior to the mons pubis. Which of the following nerves is most at risk when this incision is made? A. T10 B. T11 C. Iliohypogastric D. Ilioinguinal E. Lateral femoral cutaneous

A(. The patient has bowlegs, or genu varus. The opposite of this is genu valgus, or knock knee. The normal angle between the femoral shaft and femoral neck is between 120 and 135 degrees. In coxa vara the angle between the shaft and neck of the femur is less than 120 degrees. This can result from fractures, other injuries, or congenital softness of the bone of the femoral neck. This defect results in limb shorten- ing and limping. In coxa valga there is an increase in femoral shaft neck angulation, which can lead to hip subluxation or dislocation. Coxa valga results from weakness of the adductor musculature. Hallux valgus is commonly known as bunion. In this deformity the big toe points toward the little toe and may override the second toe; the base of the first metatarsal points medially, with a swollen bursal sac at the metatarso- phalangeal joint. Excess bony growth of the distal protruding part of the metatarsal bone can also occur. Bunions occur only rarely in people who do not rou- tinely wear shoes. GAS 554; N 476; McM 332)

65 An 81-year-old man is admitted to the emergency department with severe pain in his knees. The patient has a long history of osteoarthritis. Radiologic examina- tion reveals degeneration of the joints of his lower limbs. The degeneration is more severe on the medial side of the knees, which causes his knees to be bowed outward when he stands upright. Which of the follow- ing terms best describes the condition of his knees? A. Genu varus B. Genu valgus C. Coxa varus D. Coxa valgus E. Hallux valgus

A(. The patellar ligament is a very heavy liga- ment that connects the patella to the tibial tuberosity; it provides the insertion of the quadriceps femoris tendon upon the tibia. The patella can be thought of as a bone (a sesamoid bone) that develops within the tendon of the quadriceps femoris muscle. When the reflex hammer strikes the patellar ligament, it stretches the ligament slightly for a brief time, resulting in reflex contraction of the quadriceps femoris muscles. This reflex arc is elicited by the femoral nerve (L4 sensory input component and L2, L3 motor output). The quadriceps femoris includes the rectus femoris and the vastus lateralis, intermedius, and medialis. The patella is the largest sesamoid bone in the body. A sesamoid bone is a bone that develops within a tendon. The quadratus femoris muscle of the gluteal area arises from the ischial tuberosity and inserts on the femur proximally. The sartorius arises from the anterior superior iliac spine and inserts on the proxi- mal, medial aspect of the tibia as one of the three tendinous components of the pes anserinus (goose foot). The biceps femoris of the posterior thigh has a long head that arises from the ischial tuberosity and a short head that arises from the femur; they insert on the head of the fibula. GAS 586; N 518494 McM 335)

66 The patellar re ex appears to be markedly reduced in a 33-year-old diabetic female patient, due to deficient vascular supply of the nerves of her lower limb. The tendon of which of the following muscles is stretched during the patellar reflex? A. Quadriceps femoris B. Quadratus femoris C. Sartorius D. Pectineus E. Biceps femoris

D(. As seen in the photograph, the swollen scrotum contains mostly a clear uid. Since hydrocele is the accumulation of uid between the visceral and parietal layers of the tunica vaginalis, this condition best accounts for the ndings in this patient. GAS 470-475; N 365; McM 265)

67 A 2-year-old male toddler is admitted to the hospital with testicular pain. Physical examination reveals an enlarged scrotum. An otoscope is placed beneath the lateral side of the scrotum and the testis is transilluminated through the scrotal sac (Fig. 4-4). Which of the following best describes the signs observed in this patient? A. Varicocele B. Rectocele C. Cystocele D. Hydrocele E. Hypospadias

E(. Hallux valgus, or lateral displacement of the great toe, usually presents as pain over the prominent metatarsal head, due to rubbing from shoes, and it can be associated with deformity of the second toe, which then tends to override the great toe. Hallux valgus is commonly known as bunion. In this defor- mity the big toe points toward the little toe; the base of the first metatarsal points medially, with a swollen bursal sac at the metatarsophalangeal joint. Excess bony growth of the distal protruding part of the meta- tarsal bone can also occur. Bunions occur only rarely in people who do not routinely wear shoes. Genu varus is also referred to as bowlegs, or bandy legs, in which the knees are bowed outward. The opposite of this is genu valgus, or knock knee. The normal angle between the femoral shaft and femoral neck is between 120 and 135 degrees. In coxa vara the angle between the shaft and neck of the femur is less than 120 degrees. This can result from fractures, other injuries, or congenital softness of the bone of the femoral neck. This defect results in limb shortening and limping. In coxa valga there is an increase in femoral shaft neck angulation, which can lead to hip subluxation or dis- location. Coxa valga results from weakness of the adductor musculature. GAS 631, 634, 644; N 518; McM 350)

68 A 72-year-old man visits the outpatient clinic with a complaint of severe pain when walking. Physical examination reveals the problems in his feet as shown in Fig. 5-5. What is the most likely diagnosis? A. Coxa varus B. Coxa valgus C. Genu valgus D. Genu vara E. Hallux valgus

C(. The distal portion of the processus vaginalis contributes to the tunica vaginalis that is related to the testis. If an intermediate portion of the processus vaginalis persists, it often lls with uid, creating a hydrocele. If the entire processus vaginalis persists, the patient is likely to develop a congenital inguinal hernia. GAS 269; N 365; McM 265, 266)

7 A 3-year-old male child is admitted to the pediatric clinic. Diagnosis reveals that the intermediate portion of the processus vaginalis is not obliterated. Which of the following conditions will most likely result from this? A. Hypospadias B. Sterility C. Congenital hydrocele D. Ectopic testis E. Epispadias

D(. The piriformis muscle arises from the pelvic surface of the sacrum, passes through the greater sciatic notch, and inserts at the greater trochanter. It is considered the "anatomical key" to gluteal anatomy; the greater sciatic foramen is the "door." The gluteus medius lies posterior to the piriformis. The sciatic nerve emerges from the greater sciatic foramen, nor- mally through the infrapiriformic space. The spine of the ischium separates the greater and lesser sciatic foramina. GAS 426, 550, 575; N 490; McM 317)

7 After dividing the overlying super cial tissues and gluteal musculature in a 68-year-old female patient, the orthopedic surgeon carefully identified the underlying structures while performing a total hip arthroplasty. The key landmark in the gluteal region, relied upon in surgical explorations of this area, is provided by which of the following structures? A. Gluteus medius B. Obturator internus tendon C. Sciatic nerve D. Piriformis muscle E. Spine of the ischium

E(. In infants and children until about the age of 8 years, the head of the femur gets its arterial supply by a direct branch of the obturator artery (variably, the medial circum ex femoral). The arterial supply reaches the head of the femur at the fovea capitis by traveling along the ligament of the head of the femur. This source of supply is replaced later by vessels such as branches of the ascending branch of the medial circum ex femoral that pass into foramina of the neck of the femur within the capsule of the hip joint. Similar branches can arise from the lateral circum ex femoral and gluteal arteries. The deep circum ex iliac artery arises from the external iliac artery and supplies branches to the ilium, the iliacus muscle, and lower portions of the abdominal wall. The acetabular branch of the obturator artery often provides the branch to the head of the femur, an artery that normally regresses early in life, so that it supplies only the immediate area of the fovea capitis. The descending branch of the lateral circum ex femoral supplies the vastus late- ralis muscle and participates in anastomoses at the knee. The second perforating branch of the deep femoral artery often supplies the nutrient artery of the shaft of the femur (GAS Fig. 6-30). GAS 558-561, 676; N 491; McM 322)

72 A 75-year-old man is admitted to the emergency department with severe pain at his right hip and thigh. An MRI examination reveals avascular necrosis of the femoral head (Fig. 5-7). Which of the following arteries is most likely injured, resulting in avascular necrosis? A. Deep circumflex iliac B. Acetabular branch of obturator C. Descending branch of lateral circumflex femoral D. First perforating branch of profunda femoris E. Ascending branch of medial circumflex femoral

C(. The conjugate diameter of the pelvis (antero- posterior) is not altered by relaxation of the pelvic joints. The transverse diameter is the longest distance extending from the middle of one pelvic brim to the other. The interspinous distance is the distance between the ischial spines and changes dramatically during pregnancy due to relaxation of the joints. The diagonal conjugate and oblique diameters are slightly increased during pregnancy due to the effects of the hormone relaxin. GAS 446-453; N 332; McM 277)

74 A 23-year-old woman in her seventh month of pregnancy visits her gynecologist for a routine checkup. The patient is informed that a hormone called "relaxin" is responsible for the relaxation of the sacroiliac joint and pubic symphysis. Which of the following pelvic distances will most likely remain unaffected? A. Transverse diameter B. Interspinous distance C. True conjugate diameter D. Diagonal conjugate E. Oblique diameter

E(. The lateral plantar artery provides origin to the deep plantar arterial arch. Medially, the vascular arch anastomoses with the distal portion of the dor- salis pedis by way of the deep plantar artery. The anterior tibial artery continues as the dorsalis pedis at the ankle joint. The fibular (peroneal) artery, by way of a perforating branch in some individuals, replaces the dorsal pedis. The arcuate artery, a branch of the dorsalis pedis, provides origin for the dorsal metatarsal arteries to the lateral toes. GAS 657; N 509; McM 351)

74 A 58-year-old diabetic patient is admitted to the hospital with a painful foot. Physical examination reveals that the patient suffers from peripheral vascular disease. There is no detectable dorsalis pedis arterial pulse, but the posterior tibial pulse is strong. Which of the following arteries will most likely provide adequate collateral supply from the plantar surface to the toes and dorsum of the foot? A. Anterior tibial B. Fibular (peroneal) C. Arcuate D. Medial plantar E. Lateral plantar

C(. The bone to which the injured ligament attaches is the calcaneus. The navicular bone, located medially in the foot, articulates posteriorly with the head of the talus and anteriorly with the cuneiform bones. The cuboid bone of the lateral longitudinal arch articulates posteriorly with the calcaneus. The talus articulates with the tibia and fibula in the ankle joint mortise. GAS 633-644; N 514; McM 345)

75 A 32-year-old man is admitted to the emergency department after an injury to his foot while playing football with his college friends. An MRI examination reveals multiple tendinous tears (Fig. 5-8). Which of the following bones is associated with the muscle tears? A. Navicular B. Cuboid C. Calcaneus D. Sustentaculum tali E. Talus

D(. The obturator nerve runs a course along the lateral pelvic wall and innervates the adductors of the thigh and the skin on the medial aspect of the distal thigh. Damage to the obturator nerve is the most likely cause for the sensory and motor de cit experi- enced by the patient. The genitofemoral nerve is motor to the cremaster muscle and sensory to the skin over the femoral triangle. The ilioinguinal nerve inner- vates the skin over the labium majus and upper, inner thigh. The iliohypogastric nerve supplies skin over the anterolateral gluteal region and a strip to the area above the pubis. The lumbosacral trunk contains motor and sensory bers from L4 and L5 and is the lumbar contribution to the lumbosacral plexus. GAS 486-494, 515; N 388; McM 272)

75 A 42-year-old woman is admitted to the hospital with a mass on her right ovary. An ovariectomy is performed and the lymphatics of the lateral pelvic wall are also removed. Four days postoperatively the patient complains of painful spasms of the adductor muscles of the thigh and sensory de cit in the distal medial thigh. Which of the following nerves is most likely injured? A. Genitofemoral B. Ilioinguinal C. Iliohypogastric D. Obturator E. Lumbosacral trunk

A(. The popliteal lymph nodes are the first to receive lymph from the foot. These nodes will then drain into the deep inguinal nodes and then to the external iliac nodes. The superficial inguinal and inter- nal iliac nodes do not receive lymph from the foot. GAS 570-571; N 472; McM 340)

8 A 16-year-old boy received a superficial cut on the lateral side of his foot while playing football and is admitted to the emergency department where the wound is sutured. Four days later the patient returns to the hospital with high fever and swollen lymph nodes. Which group of nodes will first receive lymph from the infected wound? A. Popliteal B. Vertical group of superficial inguinal C. Deep inguinal D. Horizontal group of superficial inguinal E. Internal iliac

C(. The fibularis (peroneus) brevis arises from the fibula and inserts upon the tuberosity at the base of the fifth metatarsal bone. Its attachment is often involved in an inversion fracture of the foot. This common fracture can often be overlooked when it is combined with an inversion sprain of the ankle. The fibularis (peroneus) longus arises from the fibula, passes under the lateral malleolus, and then turns medially into the plantar surface of the foot, where it inserts upon the medial cuneiform and first metatarsal bones. The tibialis posterior arises from the tibia in the posterior compartment of the leg; it passes under the medial malleolus and inserts upon the navicular and metatarsal bones. The extensor digitorum brevis arises dorsally from the calcaneus and inserts upon the proximal phalanges of the lateral toes. The adduc- tor hallucis arises from the lateral metatarsals and transverse tarsal ligament and inserts upon the proxi- mal phalanx and lateral sesamoid bone of the big toe. GAS 628-629, 646, 668; N 506; McM 353)

76 An 18-year-old professional tennis player fell when she leaped for an overhead shot and landed with her foot inverted. Radiologic examination in the hospi- tal revealed an avulsion fracture of the tuberosity of the fifth metatarsal. Part of the tuberosity is pulled off, producing pain and edema. Which of the following muscles is pulling on the fractured fragment? A. Fibularis (peroneus) longus B. Tibialis posterior C. Fibularis (peroneus) brevis D. Extensor digitorum brevis E. Adductor hallucis

B(. The interspinous distance is the distance between the ischial spines. The interspinous distance is usually the shortest distance, therefore being the minimum dimension along the birth canal. The true conjugate diameter is the anteroposterior distance and does not change. The transverse diameter, oblique diameter, and diagonal conjugate diameter can change slightly during pregnancy, but the interspinous dis- tance changes the most during birth; it is also more easily measured. GAS 446-453; N 332; McM 276)

77 A 34-year-old woman is at her third stage of labor. The obstetrician is concerned that the pelvic canal is too narrow for a vaginal delivery. Which of the following dimensions is the most reliable determinant of the capacity for a vaginal birth? A. Transverse diameter B. Interspinous distance C. True conjugate diameter D. Diagonal conjugate E. Oblique diameter

A(. Excessive compression of the prepatellar bursa, as in working on bended knees, can result in pain and swelling of the prepatellar bursa, the so called housemaid's knee. Prepatellar bursitis affects plumbers, carpet layers, and other people who spend a lot of time on their knees. The bursa normally enables the patella to move smoothly under the skin. The constant friction of these occupations irritates this small lubricating sac (bursa) located just in front of the patella, resulting in a deformable tense cushion of fluid. Treatment usually requires simple drainage, but this may need to be repeated and occasionally steroids introduced. Excessive irritation of the infrapa- tellar bursa in kneeling for frequent and long periods of time (as in prayer) can result in "parson's knee." The posterior cruciate ligament of the knee can be injured in sudden, strong flexion of the knee, with posterior displacement of the tibia upon the femur. The patellar retinacula are strong, tendinous bands of tissue that join the quadriceps tendon to the vastus lateralis and medialis muscles. The lateral meniscus is a cartilaginous structure between the lateral con- dyles of the femur and tibia. GAS 606-610; N 498; McM 335)

77 A 58-year-old female employee of a houseclean- ing business visits the outpatient clinic with a com- plaint of a constant burning pain in her knees. Clinical examinations reveal a "housemaid's knee" condition (Fig. 5-9). Which of the following structures is most likely affected? A. Prepatellar bursa B. Infrapatellar bursa C. Posterior cruciate ligament D. Patellar retinacula E. Lateral meniscus

D(. The ligament of the head of the femur con- veys a small blood vessel for supply of the head of the femur (primarily in childhood). The ligament is stretched during abduction and lateral rotation of the hip joint and has an important role in stabilizing an infant's hip joint before walking. It has the potential to increase stability of the joint in hip reconstruction in developmental hip dysplasia in the pediatric popu- lation. The strength of this ligament is comparable to the anterior cruciate ligament of the knee. The iliofemoral ligament (the inverted "Y-shaped ligamentof Bigelow") on the anterior aspect of the hip bone resists hyperextension of the hip joint. The pubofemo- ral ligament arises from the pubic bone and is located on the inferior side of the hip joint; it resists abduction of the joint. The ischiofemoral ligament is a triangular band of strong bers that arises from the ischium and winds upward and laterally over the femoral neck, strengthening the capsule posteriorly. The transverse acetabular ligament attaches to the margins of the acetabular notch and provides origin for the ligament of the head of the femur. The transverse acetabular ligament is brous, not cartilaginous, but is regarded as part of the acetabular labrum. GAS 558-561; N 474; McM 325)

78 A 42-year-old mother of three children visits the outpatient clinic complaining that her youngest son cannot walk yet even though he is 4 years old. Radiologic and physical examinations reveal an unstable hip joint. Which of the following ligaments is responsible for stabilization of the hip joint in childhood? A. Iliofemoral B. Pubofemoral C. Ischiofemoral D. Ligament of the head of the femur E. Transverse acetabular ligament

B(. The posterior cruciate ligament tightens in flexion of the knee. It can be damaged by posterior displacement of the tibia upon the femur. With the patient seated, a rupture of the ligament can be dem- onstrated by the ability to push the tibia posteriorly under the femur. This is called the posterior drawer sign because it's similar to pushing in a desk drawer. The anterior cruciate ligament resists knee hyperex- tension. The lateral collateral ligament is a thick, cordlike band that passes from the lateral femoral condyle to the head of the fibula. It is located external to the capsule of the knee joint. The lateral meniscus is a nearly circular band of fibrocartilage that is located laterally within the knee joint. It is less fre- quently injured than the medial meniscus because it is not attached to the joint capsule or other ligaments. The patellar ligament is the heavy, ligamentous band of insertion of the quadriceps muscle to the tibial tuberosity. GAS 584-586; N 496; McM 332)

79 A 45-year-old is admitted to the hospital after his left leg impacted a fence post when he was thrown from a powerful four-wheel all-terrain vehicle. Radio- logic examination reveals posterior displacement of the tibia upon the femur. Which of the following structures was most likely injured? A. Anterior cruciate ligament B. Posterior cruciate ligament C. Lateral collateral ligament D. Lateral meniscus ligament E. Patellar ligament

A(. Flat foot (pes planus) is due to flattening of the medial longitudinal arch. Often congenital, it may be associated with minor structural anomalies of the tarsal bones. This condition can be seen in wet footprints in which the medial surface of the sole (normally raised in an arch) is visible. Treatment may include intensive foot exercises or arch supports worn in the shoes. Occasionally, surgery is needed in the form of arthrodesis (fusion of the tarsal bones). Pes cavus is a deformity of the foot characterized by a very high medial arch and hyperextension of the toes. The long plantar ligament is a passive ligament of the longitudinal arch. The long plantar ligament connects the calcaneus and cuboid bones. It can be involved with the plantar aponeurosis in plantar fasciitis. The long plantar ligament converts the cuboid groove into a canal for the tendon of the fibularis (peroneus) longus. The deltoid ligament is a very strong ligament that interconnects the tibia with the navicular, calca- neus, and talus bones. The medial malleolus will usually fracture before this ligament will tear. The plantar calcaneonavicular, or spring, ligament is a key element in the medial longitudinal arch; it supports the head of the talus bone and thereby is subject to vertical forces exerted through the lower limbs. In the present case, the bilateral pes planus appears to be the result of gradual weakening and failure of the arches. GAS 542, 648; N 519; McM 312)

80 A 55-year-old man visits the outpatient clinic complaining that he cannot walk more than 5 minutes without feeling severe pain in his feet. An image of the feet of this patient is shown in Fig. 5-10. What is the most common cause of this condition? A. Collapse of medial longitudinal arch, with eversion and abduction of the forefoot B. Exaggerated height of the medial longitudinal arch of the foot C. Collapse of long plantar ligament D. Collapse of deltoid ligament E. Collapse of plantar calcaneonavicular ligament

E(. The ilioinguinal nerve is a terminal branch of ventral ramus of spinal nerve L1. It innervates the skin overlying the iliac crest; the anterior portion of the urogenital region; and the upper, inner thigh. Its usual pathway takes it below McBurney's point, but it can be injured with extension of an appendectomy incision. Spinal nerve T10, the genitofemoral nerve, and the pudendal nerve are not located in the area of the incision; what is more, the area of sensory de cit does not correlate well with their injury. The genito- femoral nerve leaves the body wall at the super cial inguinal ring, well below the appendectomy incision. The pudendal nerve is both motor and sensory to the perineum. The genitofemoral nerve provides motor supply to the cremaster, sensory bers to the scrotum, and a femoral branch innervating only the skin over the femoral triangle. Spinal nerve T10 innervates the umbilical region. The subcostal nerve innervates the skin at the level of the costal margin and the lower portion of the abdominal wall above the pubic region. GAS 290, 320; N 253; McM 220)

81 A 15-year old boy underwent an appendectomy. Two weeks postoperatively the patient complains of numbness of the skin over the pubic region and anterior portion of his genitals. Which of the following nerves was most likely iatrogenically injured during the operation? A. Pudendal B. Genitofemoral C. Spinal nerve T10 D. Subcostal E. Ilioinguinal

C(. If the needle is inserted about 1.5 cm lateral to the maximal femoral pulse, it will intersect the femoral nerve in most cases. (Fluoroscopic or ultra- sound guidance is advisable to avoid iatrogenic errors.) The deep inguinal ring is located about 4 cm superolateral to the pubic tubercle and very close to the origin of the inferior epigastric vessels from the external iliac artery and vein. The approximate site of exit of the lateral femoral cutaneous nerve from the abdomen is 1.5 cm medial to the anterior superior iliac spine. Injections 1.5 cm medial to the femoral artery pulse will enter the femoral vein. Midway between the anterior superior iliac spine and the pubic symphysis can vary approximately 1.5 cm either medial or lateral from the femoral artery. GAS 566, 664, 671; N 487; McM 320)

82 A 32-year-old man is admitted to the emergency department after a car collision. Radiologic examina- tion reveals a distal fracture of the femur. The patient is in severe pain, and a femoral nerve block is admin- istered. What landmark is accurate for localizing the nerve for injection of anesthetics? A. 1.5 cm superolateral to the pubic tubercle B. 1.5 cm medial to the anterior superior iliac spine C. 1.5 cm lateral to the femoral pulse D. 1.5 cm medial to the femoral pulse E. Midway between the anterior superior iliac spine and pubic symphysis

C(. To avoid damaging the lungs, a chest tube should be placed below the level of the lungs, in the costodiaphragmatic recess. Such a point of entrance for the tube would be the eighth or ninth intercostal space. At the midclavicular line, the costodiaphrag- matic recess is localized between intercostal spaces 6 and 8, at the midaxillary line between 8 and 10, and at the paravertebral line between ribs 10 and 12. GAS 161-171, 237-239; N 214; McM 180)

83 A 34-year-old man with a complaint of sharp, localized pain over the thoracic wall is diagnosed with pleural effusion. Through which intercostal space along the midaxillary line is it most appropriate to insert a chest tube to drain the effusion fluid? A. Fourth B. Sixth C. Eighth D. Tenth E. Twelfth

D(. Inflammation of the plantar aponeurosis is referred to as plantar fasciitis. Plantar fasciitis is a common clinical condition that results from tearing or inflammation of the tough band of tissue stretching from the calcaneus to the ball of the foot (the plantar aponeurosis). It happens frequently to people who are on their feet all day, such as mail carriers, or engaged in athletics, especially in running and jumping. The pain of plantar fasciitis is usually most significant in the morning, just after you get up from bed and begin to walk. Rest, orthotics, night splints, and antiinflam- matory medications are employed in treatment. A Morton's neuroma is a painful lesion of the neural interconnection of the medial and lateral plantar nerves between the third and fourth toes. An eversion sprain of the ankle can break the medial malleolus or tear the deltoid ligament. An inversion sprain com- monly injures the fibulocalcaneal ligament or anterior talofibular ligament. GAS 651; N 519; McM 351)

84 A 34-year-old male distance runner visits the out- patient clinic with a complaint of pain he has suffered in his foot for the past week. The clinical examination indicates that the patient has an inflammation of the tough band of tissue stretching from the calcaneus to the ball of the foot. Which of the following conditions is most characteristic of these symptoms? A. Morton's neuroma B. Ankle eversion sprain C. Tarsal tunnel syndrome D. Plantar fasciitis E. Inversion sprain of the ankle

C (. The parietal pleura is innervated by the inter- costal nerves and is very sensitive to pain, in this case being somatic innervation. Therefore, the parietal pleura is the deepest layer that must be anesthetized to reduce pain during aspiration or chest tube placement. GAS 161-171; N 208; McM 184)

84 A 51-year-old man is admitted to the hospital with severe dyspnea. Radiologic examination reveals a tension pneumothorax. Adequate local anesthesia of the chest wall prior to insertion of a chest tube is necessary for pain control. Of the following layers, which is the deepest that must be infiltrated with the local anesthetic to achieve adequate anesthesia? A. Endothoracic fascia B. Intercostal muscles C. Parietal pleura D. Subcutaneous fat E. Visceral pleura

C(. The rough bony projection at the junction of the inferior end of the body of the ischium and its ramus is the large ischial tuberosity. Much of the upper body's weight rests on these tuberosities when sitting, and it provides the proximal, tendinous attach- ment of posterior thigh muscles (hamstring muscles and a portion of the adductor magnus). The small pointed posteromedial projection near the junction of the ramus and body is the ischial spine. The inferior pubic ramus is thin and at and does not bear weight. The pubic tubercle is a prominent forward projecting tubercle on the upper border of the medial portion of the superior ramus of the pubis. The iliac crest extends posteriorly, terminating at the posterior superior iliac spine. GAS 441-445; N 334; McM 286, 292)

84 After studying several hours straight in a seated position for an upcoming anatomy examination, a medical student experienced focal (nonradiating) pain in the buttocks. The pain immediately resolved when the student stood up. Which of the following bony features was most likely responsible for the student's pain? A. Inferior pubic ramus B. Posterior superior iliac spine C. Ischial tuberosity D. Pubic tubercle E. Ischial spine

B(. The child has the problem of talipes equin- ovarus, or clubfoot. Clubfoot is a congenital malfor- mation observed in about 1 in 1000 pediatric patients and rst appears in the rst trimester of pregnancy. This syndrome combines plantar exion, inversion, and adduction of the foot. The heel is drawn upward by the tendo calcaneus and turned inward; the fore- foot is also adducted, or turned inward. The foot usually is smaller than normal. In coxa vara, the angle between the femoral shaft and neck is reduced to less than 120 degrees, often due to excessive activity of the adductor musculature. Hallux valgus is also known as bunion, in which the big toe points later- ally. Hallux varus involves a medial deviation of the rst metatarsal or big toe, sometimes the result of attempted correction of bunions. It can also result from arthritis or muscular problems. GAS 534-535, 633, 660; N 514; McM 354)

85 A 5-month-old baby boy is admitted to the pedi- atric orthopedic clinic. During physical examination it is noted that the baby has inversion and adduction of the forefoot relative to the hindfoot and plantar flexion. Which of the following terms is diagnostic for the signs observed on physical examination? A. Coxa vara B. Talipes equinovarus C. Hallux valgus D. Hallux varus E. Plantar fasciitis

C(. The pelvic inlet or pelvic brim (sagittal inlet) is bounded posteriorly by the sacral promontory, lat- erally by the iliopectineal lines, and anteriorly by the symphysis pubis. The pelvic outlet (sagittal outlet) is bounded posteriorly by the coccyx, laterally by the ischial tuberosities, and anteriorly by the pubic arc. Bispinous (interspinous) outlet is the distance between the ischial spines. Maximum transverse diameter of inlet extends across the greatest width of the superior aperture, from the middle of the brim on one side to the same point on the opposite side. GAS 448-453; N 332; McM 92, 223)

85 A radiologist observed a pelvic MRI of a 22-year-old pregnant woman and measured the distance between the sacral promontory and the superior margin of the pubic symphysis. Which of the following was most likely measured by the radiologist? A. Bispinous outlet B. Maximum transverse diameter of inlet C. Sagittal inlet D. Sagittal outlet E. Bispinous inlet

D(. The ankle jerk reflex, elicited by tapping the tendo Achilles with the reflex hammer, is mediated by the tibial nerve. The superficial fibular (peroneal) nerve supplies the foot evertor muscles of the lateral compartment of the leg and provides sensory supply for the dorsum of the foot. The deep fibular (peroneal) nerve innervates the foot extensor and invertor mus- cles in the anterior compartment of the leg and sup- plies skin between the first and second toes. The common fibular (peroneal) nerve combines the func- tions of the superficial and deep branches. The medial plantar nerve innervates the abductor and flexor mus- cles of the big toe, the first lumbrical muscle, and flexor digitorum brevis muscle and provides sensation for the medial plantar surface and three and a half toes. GAS 621-625; N 516; McM 345)

87 A 62-year-old man is admitted to the emergency department. Radiologic examination and the available data indicate the likelihood of a transient ischemic attack. During physical examination the ankle jerk reflex is absent. Which of the following nerves is most likely responsible for the reflex arc? A. Common fibular (peroneal) B. Superficial fibular (peroneal) C. Deep fibular (peroneal) D. Tibial E. Superficial and deep fibular (peroneal)

B(. The posterior femoral cutaneous nerve arises from nerves S1 to S3. It provides inferior cluneal branches to the lower portion of the gluteal region and a perineal branch to the perineum and supplies sensation to the posterior thigh to the level of the popliteal fossa. Superior gluteal innervation arises from dorsal rami of L1 to L3. Meralgia paresthetica is the occurrence of pain or burning sensations on the lateral thigh, from compression of the lateral femoral cutaneous nerve. The sural nerve, sensory to the lower calf and lateral foot, arises from contributions from the tibial nerve and common fibular (peroneal) nerve. The posterior femoral cutaneous is a sensory nerve and does not innervate muscles. GAS 486, 563; N 527; McM 317)

88 Following the insertion of a prosthetic hip joint in a 72-year-old man, it was observed that the patient had greatly diminished sensation in the region of distribu- tion of the posterior femoral cutaneous nerve. Which of the following is characteristic of this nerve? A. Cutaneous supply of the superior aspect of the gluteal region B. Arises from sacral spinal nerve levels S1, S2, S3 C. Motor innervation of the obturator internus and gemelli muscles D. Injury results in meralgia paresthetica E. Provides origin of the sural nerve

C(. The patient's symptoms are all indicative of in ammatory breast cancer. Common symptoms include inversion of the nipple and dimpling of the overlying skin, changes that are due to the retraction of the suspensory ligaments (of Cooper). Intraductal cancerous tumors show symptoms including breast enlargement, breast lump, breast pain, and nipple discharge (GAS Fig. 3-16). GAS 140-141; N 179; McM 179)

88 A 55-year-old woman visited her doctor because of a painful lump in her right breast and a bloody discharge from her right nipple. Radiologic studies and physical examination reveal unilateral inversion of the nipple, and a tumor in the right upper quadrant of the breast is suspected. In addition, there is an orange-peel appearance of the skin (peau d'orange) in the vicinity of the areola. Which of the following best explains the inversion of her nipple? A. Retention of the fetal and infantile state of the nipple B. Intraductal cancerous tumor C. Retraction of the suspensory ligaments of the breast by cancer D. Obstruction of the cutaneous lymphatics, with edema of the skin E. Inflammation of the epithelial lining of the nipple and underlying hypodermis

D(. The lateral folds are key structures in forming the muscular portion of the anterior abdominal wall. Failure of the lateral folds can cause a minor defect, such as an umbilical hernia, or a major defect, such as gastroschisis. GAS 265, 311; N 247; McM 222)

9 A 28-year-old woman who is 8 months pregnant goes to the outpatient clinic for her prenatal checkup. Ultrasound examination of the fetus reveals gastroschisis, with herniation of the small bowel into the amniotic cavity. Failure of proper formation of which of the following structure(s) has resulted in this condition? A. Head fold B. Tail fold C. Neural folds D. Lateral folds E. Amnion

E(. The tibial nerve is responsible for innervating the posterior compartment of the leg. These muscles are responsible for knee flexion, plantar flexion, and intrinsic muscle functions of the foot. Compression of this nerve can affect plantar flexion of the foot. Dor- siflexion of the foot would be compromised if the deep fibular (peroneal) nerve were compressed by this Baker's cyst. Flexion of the thigh is a function of muscles supplied by lumbar nerves and the femoral nerve. The deep fibular (peroneal) nerve is also responsible for extension of the digits, whereas the femoral nerve is responsible for extension of the leg. GAS 492, 545-549, 565; N 503; McM 331)

9 A 45-year-old man presents at the local emergency clinic with the complaint of a painful knee and difficulty in walking. A computed tomography (CT) scan examination reveals a very large cyst in the popliteal fossa compressing the tibial nerve. Which movement will most likely be affected? A. Dorsiflexion of the foot B. Flexion of the thigh C. Extension of the digits D. Extension of the leg E. Plantar flexion of the foot

D(. Lymphatic drainage of the breast is typically to the axillary nodes, more speci cally to the anterior (pectoral) nodes. Lymphatic vessels from the pectoral nodes continue into the central axillary nodes, the drainage of which passes farther into the apical node, just inferior to the clavicle in the deltopectoral trian- gle. From these nodes lymph passes to the "sentinel," or scalene, nodes and the subclavian lymph trunk. The lateral and posterior axillary nodes do not nor- mally receive lymph drainage from the breast but do receive lymph from the upper limb. (This is the reason for the edema of the upper limb that occurs after a mastectomy, in which there may be a total removal of axillary lymph nodes.) GAS 140-141; N 182; McM 179)

91 A 35-year-old woman is admitted to a surgical ward with a palpable mass in her right breast and swollen lymph nodes in the ipsilateral axilla. Radiologic studies and biopsy reveal carcinoma of the breast. Which group of axillary lymph nodes is the rst to receive lymph drainage from the secretory tissue of the breast and therefore most likely to contain metastasized tumor cells? A. Lateral B. Central C. Apical D. Anterior (pectoral) E. Posterior (subscapular)

D(. Both the medial and lateral menisci are subject to rotational injuries and may be torn. The medial meniscus is much more liable to injury because it is attached to the fused deep layer of the medial collateral ligament and joint capsule. The lateral meniscus is separated from the fibular collat- eral ligament and is external to the capsule of the knee joint. Commonly seen in football players' knees, meniscal tears are usually diagnosed by MRI or by arthroscopy. The presenting symptoms of tearing may be pain and swelling, or locking of the knee. Locking of the knee suggests a bucket handle tear, in which a partly detached cartilage wedges between the tibia and femur, inhibiting further movement. A bucket handle tear is often associated with rupture of the anterior cruciate ligament. Sometimes a momentary click can be heard in flexion/extension movements of the knee. Meniscectomy is a successful operation, but currently there is greater emphasis on repairing small tears. Meniscal cysts can form secondary to meniscal tears and some of these can also be treated arthroscopically. GAS 587, 606; N 495; McM 322)

92 A 23-year-old man is admitted to the emergency department after injuring his knee while playing foot- ball. During physical examination there is pain and swelling of the knee, in addition to locking of the knee in full extension. Radiologic examination reveals a bucket handle meniscal tear (Fig. 5-12). Which of the following ligaments is most likely injured? A. Posterior cruciate B. Medial collateral C. Lateral collateral D. Anterior cruciate E. Coronary

B(. The tendons of the semitendinosus and semi-membranosus provide the superior medial border of the popliteal fossa. The semitendinosus inserts with the pes anserinus on the proximal, medial tibia. The semimembranosus inserts on the tibia posteriorly. The biceps femoris forms the superior lateral border of the fossa, as the tendon passes to insertion on the fibula. The plantaris arises from the femur just above the lateral head of the gastrocnemius, passing distally to insert on the calcaneus via the tendo Achilles. The popliteus arises from the tibia and passes superiorly and laterally to insert on the lateral condyle of the femur, with a connection to the lateral meniscus. GAS 598, 616; N 503; McM 330)

93 A 27-year-old male triathlon competitor complained that he frequently experienced deep pains in one calf that almost caused him to drop out of a regional track-and-field event. Doppler ultrasound studies indicated, and surgical exposure confirmed, the existence of an accessory portion of the medial head of the gastrocnemius that was constricting the popliteal artery. Above the medial head of the gastrocnemius, the superior medial border of the popliteal fossa could be seen. Which of the following structures forms this border? A. Tendon of biceps femoris B. Tendons of semitendinosus and semimembranosus C. Tendon of plantaris D. Adductor hiatus E. Popliteus

E(. The superior gluteal artery, which is the largest branch of the posterior division of internal iliac artery, provides spinal branches as it passes between the lumbosacral trunk and ventral ramus of S1 and exits the pelvis via the greater sciatic foramen supe- rior to the piriformis and supplying piriformis, the gluteal muscles, and tensor fasciae latae. Inferior gluteal artery, an anterior division of the internal iliac, runs inferior to the rst sacral nerve and it exits the pelvis through the infrapiriform part of the greater sciatic foramen. None of the remaining vessels tra- verses the greater sciatic foramen. GAS 495; N 380; McM 268, 276)

94 During an obstetrics and gynecology rotation, a medical student was reading a pelvic MRI of a 24-year- old woman with a physician. The physician pointed to a blood vessel that coursed between the lumbosacral trunk and ventral ramus of S1 and exited the pelvic cavity through the greater sciatic foramen. Which of the following vessels was the physician most likely pointing to? A. Inferior gluteal artery B. Superior vesical artery C. Iliolumbar artery D. Lateral sacral artery E. Superior gluteal artery

C(. The tensor fasciae latae (which is innervated by the superior gluteal nerve) and the iliotibial tract are dense, wide aponeurosis that receives the inser- tion of the tensor fasciae latae and about 75% of the gluteus maximus. The gluteus maximus is the only one of the muscles listed that is supplied by the infe- rior gluteal nerve; in fact, it is the only muscle inner- vated by the inferior gluteal nerve. Gluteus medius and minimus insert on the greater trochanter and are innervated by the superior gluteal nerve. The rectus femoris, supplied by the femoral nerve, inserts via the quadriceps tendon on the patella and tibial tuberosity. GAS 571, 664; N 481; McM 320)

94 The neurosurgeon had removed a portion of the dense tissue (dura mater) covering the brain of the patient when she removed the tumor that had invaded the skull. To replace this important tissue covering of the brain, she took a band of the aponeurotic tissue of the lateral aspect of the thigh, covering the vastus lateralis muscle. What muscle, supplied by the inferior gluteal nerve, inserts into this band of dense tissue as part of its insertion? A. Gluteus medius B. Gluteus minimus C. Gluteus maximus D. Tensor fasciae latae E. Rectus femoris

A(. The pudendal nerve passes between the sacrospinous and sacrotuberous ligaments. When performing this block with an internal approach, the ischial spine is used as a landmark as it is easily palpated transvaginally. The ischial tuberosity is used when the procedure is performed externally as it can be easily palpated in the ischioanal fossa. The posterior inferior and the posterior superior iliac spines are posterior; they cannot be palpated trans- vaginally and are not associated with the pudendal nerve. The coccyx is the most inferior portion of the vertebral column, cannot be palpated transvagi- nally, and is not near the pudendal nerve. GAS 486-494; N 389; McM 276, 277)

96 A 28-year-old pregnant woman delivers her baby before reaching the hospital and incurs a posterior vaginal tear. A pudendal nerve block is necessary to adequately anesthetize the area to facilitate proper closure of the wound. If this block is performed transvaginally, which landmark can be palpated in order to determine the proper site of anesthetic injection? A. Ischial spine B. Posterior inferior iliac spine C. Ischial tuberosity D. Posterior superior iliac spine E. Coccyx

B(. The iliopsoas forms the lateral portion of the troughlike floor of the femoral triangle. The pectineus forms the medial portion of this floor. The adductor longus provides a medial border for the femoral tri- angle and meets the sartorius, the lateral border of the triangle, at the apex. The rectus femoris is a super- ficial contributor to the quadriceps femoris, lateral to the femoral triangle (GAS Fig. 6-41). GAS 535, 572; N 487; McM 320)

96 In preparing to isolate the proximal portion of the femoral artery, the vascular surgeon gently separated it from surrounding tissues. Posterior to the femoral sheath, what muscle forms the lateral portion of the floor of the femoral triangle? A. Adductor longus B. Iliopsoas C. Sartorius D. Pectineus E. Rectus femoris

A(. The tendon of the obturator internus leaves the pelvic cavity by passing through the lesser sciatic foramen, wrapping around the lesser sciatic notch, changing direction by about 90 degrees. It is joined there by the superior and inferior gemelli and inserts with them on the upper portion of the greater trochan- ter. The obturator externus arises on the external surface of the pubic bone and obturator membrane and inserts on the greater trochanter. The quadratus femoris arises from the ischial tuberosity and inserts on the intertrochanteric line of the femur. The gluteus medius and minimus insert together on the lateral aspect of the greater trochanter. GAS 426, 448-451; N 486; McM 276)

97 A 37-year-old woman had been suffering for months from piriformis entrapment syndrome, which was not relieved by physical therapy. Part of the sciatic nerve passed through the piriformis, and a decision was made for surgical resection of the muscle. When the area of entrapment was identi ed and cleared, a tendon could be seen emerging through the lesser sciatic foramen, at rst hidden by two smaller muscles and several nerves and vessels destined for the region of the perineum. The tendons of which of the following muscles pass through this opening? A. Obturator internus B. Obturator externus C. Quadratus femoris D. Gluteus minimus E. Gluteus medius

B(. Generally, the angle of inclination between the neck and shaft of the femur in older age decreases to around 120 degrees. However, in pathologic condi- tions it can either increase or decrease from the pre- dicted value. When the angle of inclination increases, it is referred to as coxa valga. Coxa vara on the other hand is a condition characterized by a decreased angle of inclination. Genu varum and genu valgum are deformities characterized by a decreased Q-angle and increased Q-angle, respectively. The Q-angle refers to the angle between the femur and tibia. Hallux valgus is a condition that presents with a lateral devia- tion of the large toe. GAS 554, 584; N 476; McM 294)

98 A 67-year-old woman has been suffering from osteoporosis for the past year. During her annual check-up, radiologic examination reveals an angle of 160 degrees made by the axis of the femoral neck to the axis of the femoral shaft. Which of the following condi- tions is associated with these examination findings? A. Coxa vara B. Coxa valga C. Genu valgum D. Genu varum E. Hallux valgus

D(. Plantar fasciitis is a common clinical condi- tion that results from tearing or inflammation of the tough band of tissue stretching from the calcaneus to the ball of the foot (the plantar aponeurosis). It usually happens to people who are on their feet frequently or engaged in athletics, especially running and jumping. Plantar fasciitis is usually most painful in the morning, just after getting up from bed and beginning to walk. Rest, orthotics, night splints, and antiinflammatory medications are employed in treatment. A Pott's frac- ture is a bimalleolar fracture, specifically a fracture of the distal end of the fibula (lateral malleolus) and medial malleolus, with outward displacement of the foot. Dupuytren's fracture involves fracture of the distal fibula with dislocation of the foot. Each of these fractures occurs due to sudden and forceful eversion of the foot. GAS 649; N 519; McM 349)

99 A 34-year-old male runner visits the outpatient clinic complaining of pain in his foot for the past week. Physical examination reveals inflammation of the tough band of tissue stretching from the calcaneus to the ball of the foot. Which of the following conditions is char- acteristic of these symptoms? A. Pott's fracture B. Dupuytren fracture C. Tarsal tunnel D. Plantar fasciitis E. Rupture of spring ligament

B(. The parietal pleura can be divided regionally into costal, diaphragmatic, mediastinal, and cervical portions, depending upon local topographic relations. Another name for the cervical pleura is the cupula. This forms the dome of the pleura, projecting into the neck above the rst rib and corresponding to the area of injury. The costal pleura lines the internal surfaces of the ribs and intercostal spaces. The mediastinal pleura lies between the lungs and the organs in the mediastinum. The right primary bronchus and upper lobe bronchus are not in the vicinity of the right clavicle or rst rib. GAS 162-167; N 193; McM 205)

99 A 25-year-old man is admitted to the emergency department with a bullet wound in the neck just above the middle of the right clavicle and first rib. Radiologic examination reveals collapse of the right lung and a tension pneumothorax. Injury to which of the following respiratory structures resulted in the pneumothorax? A. Costal pleura B. Cupula C. Right mainstem bronchus D. Right upper lobe bronchus E. Mediastinal parietal pleura


Set pelajaran terkait

Concept Map: Bones of the Pelvic Girdle

View Set

Texas Pre-License - Real Estate Finance Practice Exam Questions

View Set

3.3.3 Quiz: Advocating Against ATOD Use

View Set

A&P Lecture: Module 5 Membrane Potential Quiz

View Set

Intervention Planning, Clinical Reasoning, Implementation, and Review

View Set